0% found this document useful (0 votes)
4 views60 pages

SAT Writing

Copyright
© © All Rights Reserved
We take content rights seriously. If you suspect this is your content, claim it here.
Available Formats
Download as PDF, TXT or read online on Scribd
0% found this document useful (0 votes)
4 views60 pages

SAT Writing

Copyright
© © All Rights Reserved
We take content rights seriously. If you suspect this is your content, claim it here.
Available Formats
Download as PDF, TXT or read online on Scribd
You are on page 1/ 60

Writing for the Digital SAT

PART 4 Introduction and Strategies


What Is Tested on the SAT Writing?
Writing When you write in academic settings and in your future career, you will need
to use correct grammar, proper wording, logical organization, and persuasive
evidence. The SAT Writing tests your skill in determining which choices lead
to the highest quality essays. In addition to your knowledge of specific
grammar rules, a broad understanding of written English will ensure you do
your very best on this section. When in doubt about your approach to the SAT
Writing, keep in mind that if you are using the same skills that would be
effective in editing a paper, you are likely doing things the right way.

How Is the SAT Writing Structured?


Part of the Reading and Writing Section (two modules of 27 questions each,
32 minutes each)
The first Reading and Writing module will be of standard difficulty, and the
second module will be more or less difficult depending on your
performance on the first module.
Approximately 25 of the 54 total Reading and Writing questions are Writing
ones.
Out of these 25 questions, about 14 relate to Standard English Conventions
and about 11 relate to the Expression of Ideas.
The writing questions come after the reading questions in a module.

SAT Writing Dos and Don’ts


Do use the full amount of time available. As you probably know from If you rush through a question like this, you will most likely make a careless
having to edit papers for school, good editing takes time. Take approximately mistake. The final sentence of the text says that the feeling of connectedness
70 seconds to complete each of the Writing questions. Some of the writing in the community “contributing” to greater mental health. In order to maintain
questions may take less time than some of the reading questions, so allocate a parallel structure with the “ing” ending, the only acceptable option would
any extra time from writing questions to the earlier reading questions. When be (B). “Contributing” and “influencing” will maintain parallelism. Choices
you evaluate more complicated Writing questions, you can make notes on the (A), (C), and (D) all change the verb structure such that a parallel style is not
provided scrap paper so that you fully understand what the question is maintained. If you rush through a question like this, you might think you are in
asking. good shape, when in fact you missed a detail essential to correctly answering
the question.
Don’t rush and make careless errors. Students are usually aware when they
have difficulty with a Math or Reading question. With SAT Writing Do look at enough context. In general, be sure to read the entirety of the
questions, it is easy to think you are doing great but to actually make a given text. If you have any doubt about whether you have sufficient context to
careless mistake. If you are still finishing the Reading and Writing section make a decision, reread the context of the text just to be sure.
with quite a bit of time left, you may want to slow down to be sure you are
getting all of the needed information and you fully understand the question Don’t only consider part of a sentence. It is easy to jump to an answer
that is being asked. This is a better use of time than having a few minutes at based on just a small part of a sentence. Keep an open mind as you read the
the end of the section to just sit there. entirety of a sentence. If you have difficulty with jumping to answers too
quickly, try covering up the answer choices until you have carefully
Example considered the relevant context.

The Roseto Effect is a concept that has expanded scientists’ Example


understanding of the effect of mental health on physical health. It has
been found that there are some areas in which certain conditions, such Marquis went to his __________ house—he was eager to visit with
as heart disease, are much less common. Scientists believe this may be both his mother and father.
due to the feeling of connectedness in the community contributing to
greater mental health and thus positively __________ physical health. Which choice completes the text so that it conforms to the conventions
of Standard English?
Which choice completes the text so that it conforms to the conventions
of Standard English? (A) parent’s
(B) parents
(A) influenced (C) parenting
(B) influencing (D) parents’
(C) has influence
(D) had influenced Unless you consider the information later in the sentence, you will have
insufficient context to determine the correct option. The second part of the
sentence states that Marquis was going to meet with “both his mother and
father.” This clarifies that he was meeting with both parents, and it is (A) application; accordingly, it
reasonable to infer that his mother and father shared the house. So, since (B) application, accordingly, it
there are two parents, show possession by putting an apostrophe after the s (C) application; accordingly it
as found in choice (D). Choice (A) would work if you had a singular parent. (D) application accordingly, it
Choice (B) would work if you were using parents as a noun instead of using
the word to show possession. And choice (C) would suggest that Marquis is Mouthing this out can help you determine where natural pauses are needed.
going to some house where he does his own parenting, which is inconsistent First, a pause is needed between the two independent clauses (complete
with the later part of the sentence. sentences)—after “application” and before “accordingly.” Second, a pause is
needed after “accordingly,” since it is an introductory word before the
Do mouth out the text as you read it. One of the best strategies for editing second independent clause. This intuitive knowledge will only get you so far.
a paper is to read it out loud in your head—that way, you will pick up on all It is important to know that a comma cannot join two independent clauses
sorts of errors that you would miss if you only quickly scanned your paper. together unless it has a conjunction like for, and, or but along with it. Also,
Many of the correct answers on the SAT Writing can be found based on how conjunctive adverbs like “accordingly” must have a semicolon precede them
things sound. In particular, punctuation, proper verb use, and parallel when they are used to join two independent clauses. So, the correct answer is
structure all lend themselves well to analysis through hearing. When taking choice (A) since it has a semicolon to separate the two independent clauses
the SAT, your job is not to go in front of a class and precisely explain the and a comma to separate the introductory word “accordingly” from the
grammatical justification for an answer. Instead, you simply need to arrive at following independent clause. Intuition and grammar knowledge together
the correct answer by any means necessary, including trusting your hearing. make for an unstoppable combination on the SAT Writing questions.

Don’t overly rely on intuition—know the rules. While hearing the passage Do use the choices to help clarify your thinking. The choices can help you
can be extremely helpful, use this approach to supplement rather than replace see where a question is headed—look at what is different among the choices
careful analysis based on grammar knowledge. Many potential grammar to determine what concept is being evaluated. If the choices use different
issues—like advanced punctuation concepts and subject-verb agreement— verbs, watch out for subject-verb agreement. If the choices use different
won’t be clear unless you understand the underlying grammar concepts. punctuation, watch for independent and dependent clauses. And if the choices
Fortunately, this book comprehensively reviews the grammar you need to have different lengths, watch for wordiness. Also, if two or more choices
know. mean essentially the same thing, that is a clue that it will not be those options.
For example, since both a semicolon and a period can be used to separate
Example two independent clauses, if you see two options that only differ in that one
uses a period and one uses a semicolon, it is likely neither of the options.
Teacher letters of recommendation are a key component of a college
__________ is important for students to develop good relationships Don’t jump to a choice too quickly. Keep an open mind as you review the
with their educators. choices. Be sure you have considered what the question is asking, what the
surrounding context says, and what the subtle differences among the choices
Which choice completes the text so that it conforms to the conventions entail. Do not treat the SAT Writing questions like ones found on a recall test
of Standard English? you may find in school, in which you can quickly remember the answer from
class. Instead, take your time.
Example proper verb tense, and correct use of punctuation. It is especially important
for students who are strong in math to keep this in mind, since they are used
I stayed up late last night studying for my test. __________ I am rather to the definitive answers found in math and sometimes feel that grammar
tired this morning. I am afraid my test results will not reflect my true rules are a matter of opinion. Keep in mind that while there are many ways
knowledge of the material. that an essay can be crafted, out of the four options given for each SAT
Writing question, there will be one and only one correct answer.
Which choice completes the text with the most logical transition?
Don’t worry about stylistic preferences and pet peeves. Here are some
(A) However, examples of pet peeves that some people have, which are not considered
(B) In contrast, widely agreed upon grammar rules:
(C) Additionally,
Using the Oxford comma—a comma before the and in a list of three or
(D) Consequently,
more items
Never starting a sentence with because
The primary way you should attack this problem is to consider the logical
Never using the more informal second person, you, and always using the
relationship between what comes before the transitional word and what
more formal third person, he, she, they, in papers
comes after it. The sentence before says that the narrator stayed up late
studying for a test, and the part after says that the narrator is now rather tired. The SAT will not test you on concepts like this because not everyone agrees
A transition that expresses a cause-and-effect relationship would be best, on them. So, you should give the SAT Writing the benefit of the doubt—
since it is because the narrator stayed up late that now the narrator is tired. instead of overthinking a question because you think it is testing some random
“Consequently” in choice (D) signifies a cause-and-effect relationship, and it rule, know that there is indeed one correct answer and look at enough context
is correct. In addition to attacking the problem this way, you can look at the until you clearly see what that correct answer is.
choices to see what clues are evident. First, all the words are transitional,
making it clear that you will need to look at what comes before and after the
Example
underlined word to determine the correct answer. Second, you can look for
similarities among the answers. “However” and “in contrast” are When preparing a college application essay, be sure to tell ______
synonymous, both meaning that what comes before and what comes after are authentic story. If you have overcome significant challenges in your life
opposites. Since these two options are synonymous, you are able to eliminate or have had interesting experiences, the application essay is a great
them both. While this strategy is likely applicable on just a handful of place to showcase your story.
problems, it can be a powerful tool if you are trying to break the tie between
a couple of options. Which choice completes the text so that it conforms to the conventions
of Standard English?
Do consider widely agreed upon grammar rules. There will be one
definite answer. The makers of the SAT ensure that when they ask a (A) one’s
question, it has been thoroughly evaluated so that it clearly has a single (B) her
answer. The SAT Writing questions will ask you about core grammatical (C) their
concepts where there is widespread agreement, like subject-verb agreement, (D) your
Students are often told to avoid using the second person, “you,” in their write organized essays. The stronger your reading and writing skills, the
writing and instead use the third person, “he, she, or they.” While this is a better you will do on the SAT Writing.
preference that many educators and writers may have, avoiding the second Practice writing in your daily life. Do not limit your writing to the
person is not technically a grammar rule. In this text, we cannot tell from the classroom. Look for opportunities to write recreationally. Instead of texting,
first sentence what word would be appropriate. It is only after examining the try sending an email with full paragraphs. Instead of only video chatting
second sentence and seeing that the writer uses “you” and “your” that we with a distant friend or relative, try writing a letter. Instead of just scanning
realize that in order to be consistent with the wording, choice (D) is correct. online articles, give your opinions in the online comments. If you are more
This is the only option that uses “you” just like the second sentence does. By ambitious, try entering writing contests, writing a blog of your own, or
realizing that there is one definite answer and that the SAT will not test you sending well-crafted correspondence to public officials with your opinions
on pet peeves, you will be able to keep an open mind as you evaluate the on important issues. Rather than waiting to the last minute to write your
choices. college application essays, begin working on them in your junior year so
that you can both write the best possible essays and improve your writing
How Can I Improve My Overall Grammar Knowledge? and editing skills.
Practice editing. Use a cloud-based word processing program so that you
Read widely. Reading will, of course, help you improve your reading
can easily have your papers edited—encourage your friends to do the same.
comprehension. However, it will also greatly help you with grammar. By
If you and your friends practice editing one another’s papers, not only will
reading high-quality books and articles, you will develop an intuitive sense
your papers be far better, but you will also sharpen the skills needed for
of what language sounds proper and what arguments are logically
success on the SAT Writing.
organized. This is especially important when building a sense of what word
choices are most appropriate. If you rely too much on a thesaurus when
coming up with synonyms, you may have difficulty identifying the word that
How Can I Use This Book to Prepare for the SAT Writing?
is best to use in a given context. Practice with the specific types of questions that are most challenging for
Write as much as you can. The more you write, the better you will be at you:
determining what arguments flow well and what organization of your essays
Subject-Verb Agreement (pages 223–228)
is most appropriate. You will have a better sense of how to avoid
Number and Tense Agreement (pages 229–231)
repetition, maintain parallel structure, and maintain subject-verb agreement.
Punctuation (pages 232–250)
When you have had to be on the lookout for good writing strategies in your
Sentence Structure and Organization (pages 251–259)
own writing, you will be much more in tune with looking for good
Transitions (pages 260–270)
strategies in the writing of other material.
Rhetorical Synthesis (pages 271–278)
Take rigorous courses that encourage you to write and read high-
quality material as much as possible. While it is tempting to take easier Practice with the Additional Practice Exercises, starting on page 278,
courses where it will not be difficult to earn an (A), this will ultimately be reviewing the types of material you will encounter on the actual test.
a disservice to you when it comes to SAT Writing questions. Take courses
like AP English Language and Composition, AP English Literature and Practice with Advanced Practice, starting on page 284, that present the most
Composition, and any of the other AP humanities courses. All of these challenging types of Writing questions you could encounter on the SAT.
courses will require you to read challenging, high-quality material and to
Practice with the full-length Reading and Writing tests, carefully watching example, “of” comes before “wolves,” with “of wolves” describing the
your time management. subject “pack.” If a word or phrase starts with a preposition, that will likely
not be the subject.
Good luck!
Example 3
Grammar Review Incorrect: The smoke detector has dead batteries, which is why they are
beeping.
Subject-Verb Agreement
Correct: The smoke detector has dead batteries, which is why it is beeping.
The SAT Writing tests your skill in determining whether nouns, pronouns, and
verbs agree numerically. A singular subject should be paired with a singular Explanation: The earlier nouns in the sentence are “detector” and “batteries.”
verb, and a plural subject should be paired with a plural verb. Which of these could most logically be described as “beeping?” It would
have to be a detector—batteries don’t beep. So, both the pronoun and the
Example 1 verb must change to be the singular “it is” instead of the plural “they are.”
Incorrect: My friend’s brand new car need to be washed.
Example 4
Correct: My friend’s brand new car needs to be washed.
Incorrect: When Darnell and Liam go to the game, he always insists on
Explanation: The subject is what is doing the action. In this case, the car is paying for the tickets.
the thing that has to be washed. Since the subject “car” is singular, it requires
Correct: When Darnell and Liam go to the game, Darnell always insists on
a singular verb, “needs.”
paying for the tickets.
Example 2 Explanation: If a pronoun is vague, it must be clarified. In this case, the “he”
could have referred to either Darnell or Liam. By revising it to have the
Incorrect: The pack of wolves are howling. person’s actual name, the sentence becomes clear. Sometimes students worry
about replacing a pronoun, thinking that they cannot be certain as to its
Correct: The pack of wolves is howling.
substitute. Do not worry about that—if a pronoun needs to be replaced
Explanation: The subject is the singular word “pack,” not “wolves.” Even because it is vague, consider the potential replacements to be true.
though there are multiple wolves within a pack, the subject is still just the
singular “pack.” “Pack” is an example of a collective noun—the word refers Example 5
to the singular group instead of the plural members of the group. Some other
examples of collective nouns include group, flock, class, herd, company, Incorrect: Everybody in the school are excited for the assembly.
and collection. Watch out for prepositions—words like “to,” “of,” “from,”
Correct: Everybody in the school is excited for the assembly.
and “in”—that typically come before a description of the subject. In this
Explanation: “Everybody” is singular, even though it refers to several 16. Anyone who loves the great outdoors (A) is or (B) are excited for good
people. Other singular pronouns include everyone, anybody, no one, weather.
someone, another, either, neither, and each. If you are unsure if a pronoun is 17. Neither terrible snow nor awful rain (A) keeps or (B) keep the mail
singular or plural, simply take the pronoun and place it next to the verb to see from being delivered.
if it makes sense. For example, you would say “anybody is here” instead of 18. Halley’s Comet, one of the solar system’s short period comets, (A) is or
“anybody are here.” (B) are projected to pass by Earth in the year 2061.
19. Since (A) it or (B) they will not be available on the trail, bring food and
Singular and Plural Agreement Practice—select the correct choice given water with you for the backpacking journey.
the context. 20. Zaha Hadid, an architect known for her parametric designs (i.e., ones
that embrace adaptive shapes and forms), (A) are or (B) is a native of
1. The food for both of my dogs (A) is or (B) are expensive. Iraq.
2. My friend Andrew forgot (A) its or (B) his lunch money.
3. (A) These or (B) This books are really interesting.
Answers
4. All the cars, including mine, (A) is or (B) are legally parked on the
street and will not be towed. 1. The food for both of my dogs (A) is expensive. “Food” is the subject
5. The flock of geese (A) are or (B) is migrating south for the winter. and is singular.
6. My English teacher and my math teacher always (A) give or (B) gives
detailed instructions in class. 2. My friend Andrew forgot (B) his lunch money. “Andrew” is a singular
7. The lamp, which has three light bulbs, (A) is or (B) are in need of some person, so “his” would be appropriate.
maintenance.
3. (A) These books are really interesting. “Books” is a plural word and
8. I discovered in my studies that while I enjoyed painting and sculpture,
“these” is used in reference to plural nouns.
(A) it was or (B) they were music I found most appealing.
9. The sights and sounds of the World’s Fair (A) were or (B) was truly 4. All the cars, including mine, (B) are legally parked on the street and
memorable. will not be towed. In this case, “all” is referring to the entire set of
10. A dog that can still walk on all fours (A) are or (B) is considered cars and is plural.
mobile.
11. When starting the car, be sure that (A) the emergency brake or (B) it is 5. 5. The flock of geese (B) is migrating south for the winter. “Flock” is a
deactivated. collective noun and is singular.
12. (A) Those or (B) This plants really have to be watered. 6. My English teacher and my math teacher always (A) give detailed
13. The captain of the team (A) has led or (B) have led her team to victory. instructions in class. There is a compound subject with both the
14. When one is speaking to one’s friends online, one should be certain that English teacher and the math teacher, so the plural verb “give” is
the chat is from one’s friends (A) themselves or (B) oneself and not needed.
from a bot.
15. The car and truck were in a close race, with (A) the one or (B) the truck 7. The lamp, which has three light bulbs, (A) is in need of some
currently in the lead. maintenance. The subject is “lamp,” not “light bulbs,” and “lamp” is a
singular word that needs the singular verb “is.”
8. I discovered in my studies that while I enjoyed painting and sculpture, 18. Halley’s Comet, one of the solar system’s short period comets, (A) is
(A) it was music I found most appealing. The singular subject “music” projected to pass by Earth in the year 2061. The verb must agree with
comes later in the sentence, but it is in fact the thing that the narrator the singular “comet,” not the plural “comets,” since it is the “comet”
finds most appealing. that will be coming by Earth in the future.
9. The sights and sounds of the World’s Fair (A) were truly memorable. 19. Since (B) they will not be available on the trail, bring food and water
There is a compound subject—sights and sounds—that requires a with you for the backpacking journey. The plural word “they” refers to
plural verb, “were.” the plural “food and water” that should go on the trail with a hiker.
10. A dog that can still walk on all fours (B) is considered mobile. The 20. Zaha Hadid, an architect known for her parametric designs (i.e., ones
subject is the singular “dog,” not the plural “fours.” Thus, the singular that embrace adaptive shapes and forms), (B) is a native of Iraq. Go
verb “is” would be correct. back to the beginning of the sentence to see that the subject is the
singular person Zaha Hadid, and not the other possible nouns like
11. When starting the car, be sure that (A) the emergency brake is
“designs,” “shapes,” or “forms.”
deactivated. In this case, there is a vague pronoun, so clarifying it by
saying “the emergency brake” would be appropriate.
Verb Agreement
12. (A) Those plants really have to be watered. The word “plants” is
plural, so the word “those” would be correct. The SAT Writing will test your skill in verb usage. You will often be asked
to determine whether a verb is used in the correct tense. Be aware of the
13. The captain of the team (A) has led her team to victory. The subject is basics of verb conjugation, as outlined in this table (you may have learned
“captain,” which is singular. A singular verb “has” is needed. quite a bit of verb conjugation in a world language course).
14. When one is speaking to one’s friends online, one should be certain
Present Past Future
that the chat is from one’s friends (A) themselves and not from a bot.
The word “themselves” refers to the word “friends,” which is plural. She is She was She will
They are They were They will
15. The car and truck were in a close race, with (B) the truck currently in I am I was I will
the lead. The pronoun would be vague in this case, so clarify with “the Present Perfect Past Perfect Future Perfect
truck.”
She has been She had been She will have been
They have been They had been They will have been
16. Anyone who loves the great outdoors (A) is excited for good weather.
I have been I had been I will have been
The word “anyone” is considered to be singular, so the singular “is”
would match.
Example 1
17. Neither terrible snow nor awful rain (A) keeps the mail from being
delivered. “Neither” and “nor” refer to objects one at a time instead of Incorrect: I went on a trip to Florida, and I have a memorable experience.
collectively, and require a singular verb; “keeps” is singular and
would work. Correct: I went on a trip to Florida, and I had a memorable experience.
Explanation: Make sure that the verbs are used consistently in the sentence. 5. You should leave a nice review for a travel guide who (A) do or (B)
Change “have” to “had” so that the sentence is all in the past tense. does a good job.
6. Before watching the sequel, I (A) watching or (B) watched the prequel.
Example 2 7. They have been swimming since early this morning and (A) is or (B)
are continuing to do so now.
Incorrect: My friend’s group has been working hard on the school project, 8. If you have the instructions, assembly of the new bicycle (A) were or
while most of the other groups are not doing so. (B) is easy.
9. It would (A) have or (B) had been an easy test if I had studied some
Correct: My friend’s group has been working hard on the school project, more.
while most of the other groups have not been doing so. 10. My neighbor will not (A) entertained or (B) entertain any new offers to
purchase his old car.
Explanation: The project has not yet been completed—it is an ongoing 11. When she flew to visit her relatives overseas, she (A) took or (B) takes
activity, so the present perfect tense is appropriate. only two flights.
12. The Renaissance artist (A) draws or (B) drew pencil sketches before he
Example 3 turned his ideas into paintings.
13. I will read my textbook this evening after I (A) had completed or (B)
In addition to ensuring that verbs are in the correct tense, the SAT Writing
complete my other homework.
will ask questions to check for both tense and number agreement.
14. An infant may have trouble falling asleep, while teenagers rarely (A)
Incorrect: When Eli lived across from two excellent parks, he have ample has or (B) have any difficulty doing so.
opportunities for outdoor recreation. 15. The guard dog barked at a predator that (A) approached or (B) will
approach the herd of sheep.
Correct: When Eli lived across from two excellent parks, he had ample 16. My father is unsure of what dish to (A) brought or (B) bring to the
opportunities for outdoor recreation. potluck dinner.
17. The flock of birds (A) have or (B) has taken off from the steep cliff to
Explanation: In the first version, “have” is incorrect with subject-verb find food.
agreement and tense. In the corrected version, “had” uses the correct past 18. Emma did a great job on her last assignment, and I am confident she (A)
tense and is numerically consistent with the subject “Eli.” will do or (B) had done well on the one due next week.
19. Jesse Owens, winner of four Olympic gold medals, (A) was or (B)
Verb Agreement Practice—select the correct choice given the context. were born in Oakville, Alabama.
20. What colleges have you (A) chose or (B) chosen for your in-person
1. Yesterday, I (A) ate or (B) eat lunch at home. visits?
2. Whenever I walk past that store, I always (A) felt or (B) feel like
making a purchase. Answers
3. My two aunts have always (A) taken or (B) took the bus to work.
4. The large boulder fell into the river and (A) diverted or (B) divert the 1. Yesterday, I (A) ate lunch at home.
water out of its typical path.
“Ate” is in the past tense, which makes sense given that the sentence Use “have” in conjunction with “would have.” It can be fine to say “if
begins with “yesterday,” putting the event in the past. it had,” but do not use the word “would” along with “had.”

2. Whenever I walk past that store, I always (B) feel like making a 10. My neighbor will not (B) entertain any new offers to purchase his old
purchase. car.
“Feel” is in the present tense, consistent with the earlier present tense “Will not entertain” is the proper conjugation to express that this will
verb “walk.” take place in the future; “entertained” is fine as a past tense option,
but not along with “will not.”
3. My two aunts have always (A) taken the bus to work.
“Have taken” is the correct conjugation, while “have took” would not 11. When she flew to visit her relatives overseas, she (A) took only two
work; “took” by itself is fine to express the past tense, but not in flights.
conjunction with “have.” “Took” is consistent with the earlier past tense verb “flew.”

4. The large boulder fell into the river and (A) diverted the water out of 12. The Renaissance artist (B) drew pencil sketches before he turned his
its typical path. ideas into paintings.
The earlier verb “fell” is in the past tense, so use “diverted” to be Look later in the sentence to see that it is in the past tense given the
consistent throughout the sentence. verb “turned”; therefore, use the past tense “drew” for consistency.

5. You should leave a nice review for a travel guide who (B) does a 13. I will read my textbook this evening after I (B) complete my other
good job. homework.
The noun “guide” is singular and therefore requires a singular verb, Since the textbook reading is happening in the future after the
“does.” completion of the homework, it is appropriate to use the present tense
for “complete.”
6. Before watching the sequel, I (B) watched the prequel.
Using “watched” is logical given that this is a past series of events; 14. An infant may have trouble falling asleep, while teenagers rarely (B)
also, using “watching” would prevent this from being a complete have any difficulty doing so.
sentence. “Have” will be consistent with the plural subject “teenagers.”

7. They have been swimming since early this morning and (B) are 15. The guard dog barked at a predator that (A) approached the herd of
continuing to do so now. sheep.
“Are” is consistent with the earlier subject “they.” “Approached” is consistent with the past tense verb “barked” earlier
in the sentence.
8. If you have the instructions, assembly of the new bicycle (B) is easy.
“Is” will keep the sentence in the present tense, and it will be 16. My father is unsure of what dish to (B) bring to the potluck dinner.
consistent with the singular subject “assembly.” “To bring” is the correct infinitive form of the verb, not “to brought.”

9. It would (A) have been an easy test if I had studied some more. 17. The flock of birds (B) has taken off from the steep cliff to find food.
The subject is the singular noun “flock,” so “has” would be consistent Which choice completes the text so that it conforms to the conventions
with it. of Standard English?

18. Emma did a great job on her last assignment, and I am confident she (A) blue team won the game.
(A) will do well on the one due next week. (B) game was won by the blue team.
Based on context clues, the correct verb will be “will do” since the (C) winning by the blue team was completed.
next assignment is one that has not yet been evaluated. (D) blue team winning of the game was done.

19. Jesse Owens, winner of four Olympic gold medals, (A) was born in 3. A variety of diplomats gathered at the conference to discuss the new
Oakville, Alabama. treaty. Out of the officials present, the Secretary General of the United
The subject is the singular person “Jesse Owens,” not the plural Nations _____ the most well-respected.
“medals”; therefore, use the singular verb “was.”
Which choice completes the text so that it conforms to the conventions
20. What colleges have you (B) chosen for your in-person visits? of Standard English?
“Have chosen” is the correct conjugation of the verb, not “have
chose.” (A) will
(B) were
Number and Tense Agreement Practice Questions (C) are
(D) was
1. Anyone who assembles comic book collections probably _______
superheroes. After all, Batman, Superman, Wonder Woman, and
4. Three years ago, she went on vacation and _______ her relatives. Now
Spiderman are some of the most popular characters found in these
she is going to do a similar trip with her family but plans to go to a
books.
different destination.
Which choice completes the text so that it conforms to the conventions
Which choice completes the text so that it conforms to the conventions
of Standard English?
of Standard English?
(A) enjoy
(A) will visit
(B) enjoying
(B) visited
(C) enjoys
(C) visiting
(D) to enjoy
(D) visit
2. The two hockey teams were evenly matched—throughout the game, the
5. The three friends were excited to meet up for a stroll in the new city
team that was ahead changed many times. After a hard-fought contest,
park. Jennifer and Debby couldn’t wait to see _____ new dog.
the __________________. Ultimately their goalie’s defensive prowess
proved pivotal.
Which choice completes the text so that it conforms to the conventions Which choice completes the text so that it conforms to the conventions
of Standard English? of Standard English?

(A) her (A) has


(B) their (B) have
(C) Pam’s (C) had
(D) whose (D) having

6. Now that we have started the new employee training, I must make my 9. Make sure you are ready the day before the SAT with everything that
expectations about tardiness very clear. I require that everyone ____ you will need. Because _______ not provided at the testing center, a
punctual. If you cannot show up on time for the job, you might as well bottle of water and a snack are two things you should take with you.
not show up at all.
Which choice completes the text so that it conforms to the conventions
Which choice completes the text so that it conforms to the conventions of Standard English?
of Standard English?
(A) they are
(A) is (B) it is
(B) be (C) you are
(C) are (D) we are
(D) was
10. We look out the window of our apartment and can see many things
7. When you are looking for a new job, there are a few things to keep in happening. There are people walking on the sidewalk and kids playing
mind. You should work on your networking skills, _______ resume, on the playground. Also, ____________________________
and your references.
Which choice completes the text so that it conforms to the conventions
Which choice completes the text so that it conforms to the conventions of Standard English?
of Standard English?
(A) the cars driving down the freeway.
(A) your (B) the cars are driving down the freeway.
(B) one’s (C) the cars are drive down the freeway.
(C) you’re (D) the cars down the freeway driving.
(D) its
Answer Explanations
8. My friend and I went on a hike around the lake. We saw fish, plants, and
plenty of birds. In fact, there was a flock of geese that ____ at least 30 1. (C) The subject is “anyone,” which is singular—it refers to each person
birds. one at a time. “Enjoys” is the only verb that would work to provide a
complete sentence that aligns with a singular subject. Punctuation
2. (A) This option puts the words in the most logical order, making it clear Commas
that the blue team won the game. Choices (B) and (C) use the passive
voice with “by.” Choice (D) has a convoluted word order. When it comes to comma questions on the SAT, it is important to know both
when to use a comma and when NOT to use a comma. You may be able to
3. (D) This verb aligns with the singular person of the “Secretary intuitively tell when to use a comma by mouthing out the sentence in your
General,” so the singular verb “was” works. Also, it is in the past head and “hearing” where a small breath or pause would be. However, it is
tense, consistent with the earlier past tense verb “gathered.” The other helpful to know specific rules about when and when not to use commas so
options are either in the incorrect tense, like “will,” or plural, like that you can do your best.
“were” and “are.”
Do use a comma to separate an introductory phrase (dependent clause)
4. (B) The verb “went” earlier in the sentence is in the past tense, and from a complete sentence (independent clause).
“visited” is the only option that is also in the past tense.
Correct: Whenever my brother sleeps, he snores loudly.
5. (C) Pronouns are perfectly fine to use so long as what they represent is
clear. In this sentence, it is unclear who possesses the dog unless the Correct: If you are happy with your grade, put the paper on your refrigerator.
owner is specifically named as in choice (C). The other options would
leave the ownership unclear. Explanation: In both of these examples, there is an introductory phrase
followed by a complete sentence. A comma is great for separating dependent
6. (B) Since this is a demand, the verb “be” is appropriate. The other clauses (incomplete thoughts that cannot stand on their own) from complete
options would not be typically used in a command. sentences (a complete idea with both a subject and a verb).
7. (A) This is the only option that maintains parallelism with the other Don’t use a comma by itself to join two complete sentences (independent
parts of the list—they all begin with “your.” clauses).
8. (C) “Had” is consistent with the past tense verb “was” earlier in the Incorrect: Jean went for a walk around town, she saw many different
sentence. The other options are not in the past tense. neighborhoods.
9. (A) To see what pronoun is needed, look at what comes later in the Correct: Jean went for a walk around town, and she saw many different
sentence. A watch and a snack are the things that are not provided, so neighborhoods.
in reference to these two things, “they” is the only possibility.
Incorrect: My friend loves to paint, he makes large messes when doing so.
10. (B) This option puts the words in a logical sequence and provides a
complete sentence by saying “cars are driving.” Choices (A) and (D) Correct: My friend loves to paint, but he makes large messes when doing so.
result in a sentence fragment. Choice (C) should use “driven” instead
of “drive” in conjunction with “are.”
Explanation: A comma by itself cannot separate one complete sentence from Incorrect: The book, that I have almost finished is very suspenseful.
another—this error is called a comma splice. If you had a word like and,
but, or yet along with the comma, that would be fine. Also, a semicolon, Correct: The book that I have almost finished is very suspenseful.
period, and sometimes a dash or colon can work to separate complete
sentences. Explanation: These sentences would work if the commas were removed. Just
because a sentence has a long subject does not mean that it requires a comma.
Do use a comma before and after a parenthetical phrase (i.e., words that Note that phrases that do not require a comma will often start with that. That
are added to a sentence without changing the original sentence’s indicates that a phrase is restrictive, or essential, to the sentence’s meaning.
grammar or meaning).
Do use a comma to separate words within a list.
Correct: The first person to walk on the moon, Neil Armstrong, is a native of
the state of Ohio. Correct: I will pick up apples, bananas, and oranges at the fruit stand.

Correct: Jigsaw puzzles, unlike crossword puzzles, involve visual pattern Don’t use a comma right before a list begins.
recognition.
Incorrect: At the fruit stand, I will pick up, oranges, apples, and bananas.
Correct: A box of my favorite cereal, which I purchased on sale, is what my
Correct: At the fruit stand, I will pick up oranges, apples, and bananas.
family will have for breakfast this week.
Explanation: There is no need for a comma after “pick up” in the second
Explanation: The phrases “Neil Armstrong,” “unlike crossword puzzles,”
sentence. Just use commas to separate the items within the list.
and “which I purchased on sale” could be removed from the original
sentences and the sentences would remain complete with their original Note: The SAT uses the Oxford comma with lists: the comma before the and
meanings. Note that parenthetical phrases will often start with which. (If that comes before the final item. However, since the Oxford comma is more
which is used without a parenthetical phrase, like “the building in which I of a stylistic choice than a grammar rule, they most likely will not test you on
will take the SAT,” no comma before which is needed). this concept.
Don’t use a comma to separate the subject from the verb in a sentence. Do use a comma along with coordinating conjunctions to connect two
complete sentences. Coordinating conjunctions provide a strong
Incorrect: My best friend, is moving out of town.
connection between sentences. Remember coordinating conjunctions
Correct: My best friend is moving out of town. with the acronym FANBOYS (For, And, Nor, But, Or, Yet, So).

Incorrect: Football players and basketball players, both use locker rooms to Correct: My sister was hungry, so she grabbed a snack from the kitchen.
get ready.
Correct: I studied quite a bit for the test, but I did not perform as well as I
Correct: Football players and basketball players both use locker rooms to get would have liked.
ready.
Don’t just use a comma to connect two complete sentences joined by a Explanation: It does not change the meaning of the sentence whether “fluffy”
conjunctive adverb (e.g., however, indeed, nevertheless, moreover, or “friendly” comes first.
namely, meanwhile, subsequently, thus, furthermore). Conjunctive
adverbs provide a weaker connection between sentences. Instead, use a Don’t use commas to separate adjectives if the order of the adjectives
semicolon with a comma. does matter.

Incorrect: He was delighted to be promoted, however, he was not happy with Incorrect: The first, female governor won a large majority of the vote.
the new work schedule.
Incorrect: The female, first governor won a large majority of the vote.
Correct: He was delighted to be promoted; however, he was not happy with
the new work schedule. Correct: The first female governor won a large majority of the vote.

Incorrect: The tour started at the museum, subsequently, it went to the park. Explanation: The order of the adjectives “first” and “female” does matter
—“first” must come before “female” in order for the sentence to be logical.
Correct: The tour started at the museum; subsequently, it went to the park. In this situation, do not use commas to separate the adjectives.

Note: It is fine to use conjunctive adverbs with just commas so long as they Comma Practice—select the better sentence out of the two options.
are not used to connect two complete sentences:
1. (A) Once upon a time, the kingdom was under attack.
Correct: My friend wanted me to come to his house. Instead, I decided to stay (B) Once upon a time the kingdom was under attack.
home.
2. (A) My Advanced Placement World History class, is very interesting.
Correct: Most birds can fly. The penguin, however, cannot. (B) My Advanced Placement World History class is very interesting.

Explanation: While a comma by itself is not enough to join two complete 3. (A) I needed a quiet place to read, so I went to the library.
sentences, a comma along with one of the FANBOYS words will be enough. (B) I needed a quiet place to read so I went to the library.
Be sure that the word joining the two complete sentences is one of these
4. (A) The pedestal onto, which you place the trophies should be sturdy.
FANBOYS words—if it is one of the conjunctive adverbs, use a semicolon
(B) The pedestal onto which you place the trophies should be sturdy.
before the conjunctive adverb and a comma after the conjunctive adverb to
make the connection. 5. (A) If you are going to do well on your test, be sure to get a good night’s
sleep.
Do use commas to separate adjectives if the order of the adjectives does (B) If you are going to do well on your test be sure to get a good night’s
not matter. sleep.
Correct: The fluffy, friendly puppy likes to wag its tail. 6. (A) One example of a large animal, is the blue whale.
(B) One example of a large animal is the blue whale.
Correct: The friendly, fluffy puppy likes to wag its tail.
7. (A) The first person to walk across Antarctica without any assistance 16. (A) Our house, which we moved into last year, is still in good
Colin O’Brady took 54 days to do so. condition.
(B) The first person to walk across Antarctica without any assistance, (B) Our house which we moved into last year is still in good condition.
Colin O’Brady, took 54 days to do so.
17. (A) The job of a lifeguard, is to vigilantly keep watch over the beach.
8. (A) Essential ingredients for the cake are flour, sugar, and eggs. (B) The job of a lifeguard is to vigilantly keep watch over the beach.
(B) Essential ingredients for the cake are flour sugar and eggs.
18. (A) You did an excellent job on your group project; moreover, you aced
9. (A) The results of the study, indicate that the medicine is highly your final exam.
effective. (B) You did an excellent job on your group project, moreover, you aced
(B) The results of the study indicate that the medicine is highly your final exam.
effective.
19. (A) My newest neighbor Jian, loves to play basketball in his driveway.
10. (A) The saw is extremely precise, cutting each edge flawlessly. (B) My newest neighbor, Jian, loves to play basketball in his driveway.
(B) The saw is extremely precise cutting each edge flawlessly.
20. (A) If you are passionate about your profession, work will not feel like
11. (A) Broccoli, though not my friend’s favorite, is something I love to eat “work.”
whenever I can. (B) If you are passionate about your profession work will not feel like
(B) Broccoli though not my friend’s favorite is something I love to eat “work.”
whenever I can.
21. (A) The new volleyball coach, was excited for the team’s first game.
12. (A) The chair that is by the window is extremely comfortable. (B) The new volleyball coach was excited for the team’s first game.
(B) The chair, that is by the window, is extremely comfortable.
22. (A) Cold-brewed coffee, though not my personal favorite, has become
13. (A) Each morning, I like to take a shower, eat breakfast, and gather my increasingly popular.
things for school. (B) Cold-brewed coffee, though not my personal favorite has become
(B) Each morning, I like to take a shower eat breakfast, and gather my increasingly popular.
things for school.
23. (A) The traffic, that clogged up the highway for eight straight miles, was
14. (A) Inflation is largely attributable to the ever-increasing prices of quite a nuisance.
different goods and services. (B) The traffic that clogged up the highway for eight straight miles was
(B) Inflation is largely attributable, to the ever-increasing prices of quite a nuisance.
different goods and services.
24. (A) My videoconferencing software which I just updated helped me to
15. (A) The ping-pong table in the middle of the gymnasium, is where I meet with faraway relatives.
spend my time after eating lunch. (B) My videoconferencing software, which I just updated, helped me to
(B) The ping-pong table in the middle of the gymnasium is where I meet with faraway relatives.
spend my time after eating lunch.
25. (A) Several thousand years ago, people, along with many animals, 9. (B) The results of the study indicate that the medicine is highly
traversed an ice bridge from Eurasia to the Americas. effective.
(B) Several thousand years ago people, along with many animals There is no need to separate the subject and the verb with a comma.
traversed an ice bridge from Eurasia to the Americas.
10. (A) The saw is extremely precise, cutting each edge flawlessly.
Answers Use a comma to separate the complete sentence from the dependent
clause that follows.
1. (A) Once upon a time, the kingdom was under attack.
Place a comma after the introductory phrase. 11. (A) Broccoli, though not my friend’s favorite, is something I love to eat
whenever I can.
2. (B) My Advanced Placement World History class is very interesting. Surround the parenthetical phrase with commas.
Don’t separate the subject from the verb with a comma.
12. (A) The chair that is by the window is extremely comfortable.
3. (A) I needed a quiet place to read, so I went to the library. The is no need for commas when using “that” to describe something
“So” is one of the FANBOYS words, and can therefore be used to join essential to the subject.
two sentences along with a comma.
13. (A) Each morning, I like to take a shower, eat breakfast, and gather my
4. (B) The pedestal onto which you place the trophies should be sturdy. things for school. Use commas to separate the items in the list.
This is an example of using “which” when it does not precede a
parenthetical phrase—thus, no comma is needed. 14. (A) Inflation is largely attributable to the ever-increasing prices of
different goods and services.
5. (A) If you are going to do well on your test, be sure to get a good Do not use a comma to break up the phrase “attributable to.”
night’s sleep.
Use a comma to separate the introductory phrase from the complete 15. (B) The ping-pong table in the middle of the gymnasium is where I
sentence that follows. spend my time after eating lunch.
Even though there is a long subject in the sentence, do not use a comma
6. (B) One example of a large animal is the blue whale. to separate the subject from the verb.
There is no need to separate the subject and the verb.
16. (A) Our house, which we moved into last year, is still in good
7. (B) The first person to walk across Antarctica without any assistance, condition.
Colin O’Brady, took 54 days to do so. Use commas around the parenthetical phrase that begins with “which.”
Surround the name with commas since the sentence will maintain its
meaning and grammar without it. 17. (B) The job of a lifeguard is to vigilantly keep watch over the beach.
Do not separate the subject from the verb.
8. (A) Essential ingredients for the cake are flour, sugar, and eggs.
Use commas to separate the items in the list. 18. (A) You did an excellent job on your group project; moreover, you aced
your final exam. Use a semicolon, not a comma, to separate two
sentences joined by a conjunctive adverb. cases, using the semicolon allows for more stylistic variety in one’s writing.

19. (B) My newest neighbor, Jian, loves to play basketball in his driveway. Correct: I am excited to go to the concert; I will show up early to be first in
“Jian” is the same person as “my newest neighbor,” so his name can be line.
set off with commas.
Incorrect: While I am excited to go to the concert; there is no way I want to
20. (A) If you are passionate about your profession, work will not feel like show up early to wait.
“work.”
Use commas to separate the introductory phrase from the complete Correct: While I am excited to go to the concert, there is no way I want to
sentence that follows. show up early to wait.

21. (B) The new volleyball coach was excited for the team’s first game. Explanation: If there are complete sentences on either side of the semicolon,
Do not separate the subject from the verb. it is appropriate to use one. If there is a sentence fragment on either side of
the semicolon, change the sentence structure (perhaps by substituting a
22. (A) Cold-brewed coffee, though not my personal favorite, has become comma for the semicolon) to make it work.
increasingly popular. Surround the parenthetical phrase with commas.
Use a semicolon to separate items in a list when each item has internal
23. (B) The traffic that clogged up the highway for eight straight miles was punctuation, like a comma, within it.
quite a nuisance.
No commas are needed to surround an essential phrase starting with Correct: When I tour colleges, I am looking forward to checking out Boston,
“that.” Massachusetts; Ithaca, New York; and Providence, Rhode Island.

24. (B) My videoconferencing software, which I just updated, helped me to Incorrect: When I tour colleges, I am looking forward to checking out Boston,
meet with faraway relatives. Massachusetts, Ithaca, New York, and Providence, Rhode Island.
Surround the parenthetical, nonessential phrase that begins with
“which” with commas. Explanation: The semicolons create a clear distinction between each place
that is denoted as a city and state. Without the semicolons to separate these
25. (A) Several thousand years ago, people, along with many animals, items, it would seem like the cities and states were different things.
traversed an ice bridge from Eurasia to the Americas.
Use a comma after “people” to separate the introductory phrase, and Semicolon Practice—select the better sentence out of the two options.
also use commas around the parenthetical phrase “along with many
1. (A) This is the best book I have ever read, I simply cannot put it down.
animals.”
(B) This is the best book I have ever read; I simply cannot put it down.
Semicolons 2. (A) A positive attitude can help you overcome adversity, no matter the
circumstances.
Use a semicolon (;) to separate two complete, related sentences from
one another. While a period could be substituted for a semicolon in most
(B) A positive attitude can help you overcome adversity; no matter the A comma will work to separate the introductory phrase from the
circumstances. complete sentence.

3. (A) It was supposed to be a cloudy day, however, the weather forecast Colons
was fortunately incorrect.
(B) It was supposed to be a cloudy day; however, the weather forecast Use a colon (:) if it comes after a complete sentence and introduces a
was fortunately incorrect. clarification or a list. If an idea is not fully expressed, the colon can set off
the information that will complete the thought.
4. (A) The most critical people on the movie set are John, the director;
Susan, the producer; and Harris, the lead actor. Correct: After many years of hard work, I have achieved my lifelong dream:
(B) The most critical people on the movie set are John, the director, hiking up a tall mountain.
Susan, the producer, and Harris, the lead actor.
Correct: French mirepoix uses these ingredients: carrots, onions, and celery.
5. (A) If the grass is dry, water the lawn.
(B) If the grass is dry; water the lawn. Colon Practice—select the better sentence out of the two options.

Answers 1. (A) My houseplant requires the following to thrive sunlight, fertilizer,


and water.
1. (B) This is the best book I have ever read; I simply cannot put it down. (B) My houseplant requires the following to thrive: sunlight, fertilizer,
Use a semicolon to separate the two complete sentences. and water.

2. (A) A positive attitude can help you overcome adversity, no matter the 2. (A) The office workers finally agreed on the setting for the thermostat:
circumstances. 72 degrees.
“No matter the circumstances” is not a complete sentence, so separate it (B) The office workers finally: agreed on the setting for the thermostat
from the complete sentence that comes before it with a comma. 72 degrees.

3. (B) It was supposed to be a cloudy day; however, the weather forecast 3. (A) You should take: your suitcase, backpack, and wallet with you on
was fortunately incorrect. the trip.
Recall that with conjunctive adverbs (like “however”) that join two (B) You should take your suitcase, backpack, and wallet with you on the
complete sentences, use a semicolon followed by a comma. trip.

4. (A) The most critical people on the movie set are John, the director; 4. (A) We were delighted to see two of our favorite animals at the zoo:
Susan, the producer; and Harris, the lead actor. elephants and tigers.
Use semicolons to separate the items in the list that have internal (B) We were delighted to see two of our favorite animals at the zoo
punctuation. elephants and tigers.

5. (A) If the grass is dry, water the lawn.


5. (A) No matter how you choose to season your food, salt and pepper are Correct: I would not go down that alley—it looks very spooky.
recommended items to have on a dinner table.
(B) No matter how you choose to season your food: salt and pepper are Correct: Wait—I am not ready to go.
recommended items to have on a dinner table.
Use dashes to surround a parenthetical phrase. Start and finish the
Answers parenthetical phrase using the same punctuation, like commas, dashes, or
parentheses.
1. (B) My houseplant requires the following to thrive: sunlight, fertilizer,
Correct: Las Vegas—a major tourist town—is surprisingly surrounded by
and water.
desert.
The colon comes before the list of items the plant requires to thrive.
Incorrect: Las Vegas—a major tourist town, is surprisingly surrounded by
2. (A) The office workers finally agreed on the setting for the thermostat:
desert.
72 degrees.
This is the correct placement for the colon since it comes after a Note: With the above examples, other forms of punctuation could have been
complete sentence and before a clarification. used instead of the dash. You have a great deal of flexibility in your
punctuation choices in more complex sentences. However, do not worry
3. (B) You should take your suitcase, backpack, and wallet with you on
about whether the SAT will have two correct options. If a colon could be
the trip.
used for a dash and vice versa, only one of these options will be presented.
Not every list will require a colon to come before it. In this case, a
colon would cause a very abrupt pause. Dash Practice—select the better sentence out of the two options.
4. (A) We were delighted to see two of our favorite animals at the zoo: 1. (A) My favorite food—pizza—is easily found at many restaurants.
elephants and tigers. The colon comes before the clarification of which (B) My favorite food, pizza—is easily found at many restaurants.
animals were at the zoo.
2. (A) The backpacking journey looks quite challenging—be prepared.
5. (A) No matter how you choose to season your food, salt and pepper are (B) The backpacking journey looks quite challenging be prepared.
recommended items to have on a dinner table.
Since “No matter how you choose to season your food” is not a 3. (A) My science teacher—a former government researcher—is excellent
complete sentence, having a colon after it would not work. Use a at helping us see the applications of what we learn.
comma instead. (B) My science teacher—a former government researcher is excellent at
helping us see the applications of what we learn.
Dashes
4. (A) Based on the latest data, the geologists could only come to one
Use a dash (—) to indicate a change of thought or change of voice in a conclusion—a volcanic eruption was imminent.
sentence. (B) Based on the latest data, the geologists could only come to one
conclusion a volcanic eruption was imminent.
5. (A) My friends Sharon and Maria are coming over to my house this Caitlin’s shoes
evening. My car’s steering wheel
(B) My friends Sharon—and Maria—are coming over to my house this
Use an apostrophe after an s to show plural possession.
evening.
Three pets’ fence
Answers Seven cars’ parking lot spaces
Our books’ shelf
1. (A) My favorite food—pizza—is easily found at many restaurants. Three families’ neighborhood
Be consistent in using the same type of punctuation on either side of
parenthetical phrases. Note: It does not matter how many things are possessed/owned; it matters
how many owners there are.
2. (A) The backpacking journey looks quite challenging—be prepared.
Have a dash toward the end of the sentence to indicate the change of Less Common Situations
thought.
Use an apostrophe before an s to show possession if the word is already
3. (A) My science teacher—a former government researcher—is excellent plural.
at helping us see the applications of what we learn. The women’s organization
Surround the clarification about the science teacher’s background with The children’s playground
dashes.
Use just one apostrophe if both of the nouns in a compound subject
4. (A) Based on the latest data, the geologists could only come to one jointly own something.
conclusion—a volcanic eruption was imminent. Pam and Andy’s family (they share a family)
Have a dash before the clarification of what the conclusion was. The cat and dog’s toys (they share the toys)
5. (A) My friends Sharon and Maria are coming over to my house this Use apostrophes after each of the nouns in a sequence if each noun
evening. individually possesses an item.
There is no need for dashes in this situation because there is no
Aliyah’s and Olivia’s report cards were each excellent. (Aliyah and Olivia
parenthetical phrase. Instead, “Maria” is part of the subject.
have their own report cards.)
My sister’s and my brother’s rooms are both messy. (The sister and brother
Apostrophes individually have messy rooms.)
Common Situations Note: Do not worry about apostrophe placement exceptions for names
ending in s. The SAT will stick to widely agreed upon apostrophe rules.
Use an apostrophe (’) before an s to show singular possession.
One pet’s dish TIP
A person’s house
Another way to keep these rules straight is to realize that if you remove the apostrophe and what comes 5. (A) Humpback whales love to swim in the open ocean.
after it, you will get the original non-possessive word.
(B) Humpback whales’ love to swim in the open ocean.

6. (A) The Broadway show’s sets are difficult to move.


For example: (B) The Broadway show’s set’s are difficult to move.
Possessive Form What to Remove Original Word 7. (A) When the new coach helped the men’s soccer team, he made
Dog’s ’s Dog significant adjustments to the lineup.
Friends’ ’ Friends (B) When the new coach helped the mens’ soccer team, he made
Men’s ’s Men significant adjustments to the lineup.

8. (A) As you read the joint paper by the scientists, note how Watson’s and
Apostrophes and Pronouns Crick’s results match their hypothesis.
Do use an apostrophe on pronoun contractions—ones that have a (B) As you read the joint paper by the scientists, note how Watson and
pronoun and verb together. Crick’s results match their hypothesis.
It’s a beautiful day outside. (same as “it is”) 9. (A) His backpack needs it’s straps repaired.
They’re coming over this evening. (same as “they are”) (B) His backpack needs its straps repaired.
You’re doing well with your homework. (same as “you are”)
10. (A) The three farmers’ fields encompassed over 500 acres.
Don’t use an apostrophe with pronouns that show possession. (B) The three farmers fields encompassed over 500 acres.
The bicycle needs its tires inflated. (The tires belong to the bicycle.)
Their clothes are now clean. (The clothes belong to the implicit people.) Answers
You should mind your manners at dinner. (The manners belong to you.)
1. (B) One person’s trash is another person’s treasure.
Apostrophe Practice—select the better sentence out of the two options. Since it is just one person who possesses the trash, have the apostrophe
before the s.
1. (A) One persons’ trash is another person’s treasure.
(B) One person’s trash is another person’s treasure. 2. (B) The teachers are meeting to share their best lesson ideas.
2. (A) The teachers’ are meeting to share their best lesson ideas. No apostrophe is needed since the teachers are not showing possession.
(B) The teachers are meeting to share their best lesson ideas.
3. (A) The natural history museum’s planetarium is extremely impressive.
3. (A) The natural history museum’s planetarium is extremely impressive. The planetarium belongs to the singular museum.
(B) The natural history museums planetarium is extremely impressive.
4. (A) Whenever my relatives come over, we need to order extra food.
4. (A) Whenever my relatives come over, we need to order extra food. The relatives are doing the action of coming over, not possessing
(B) Whenever my relatives’ come over, we need to order extra food. anything, so no apostrophe is needed.
5. (A) Humpback whales love to swim in the open ocean. In all the above examples, the parentheses do not affect the usage of the
The whales are acting as a subject, not possessing anything, so no commas and periods that surround them.
apostrophe is required.
Parentheses Practice—select the better sentence out of the two options.
6. (A) The Broadway show’s sets are difficult to move.
The sets belong to the show; the sets do not possess anything. 1. (A) The large dog eats quite a bit of food each day (mainly food from a
bag).
7. (A) When the new coach helped the men’s soccer team, he made (B) The large dog eats quite a bit of food each day (mainly food from a
significant adjustments to the lineup. bag.)
Since “men” is already plural, put the apostrophe before the s.
2. (A) When you eat breakfast (considered the most important meal of the
8. (B) As you read the joint paper by the scientists, note how Watson and day,) you have energy for several hours.
Crick’s results match their hypothesis. (B) When you eat breakfast (considered the most important meal of the
Since Watson and Crick collectively own the results, just put an day), you have energy for several hours.
apostrophe before the s in the final name.
3. (A) Cricket (once an Olympic game,) is one of my favorite sports.
9. (B) His backpack needs its straps repaired. (B) Cricket (once an Olympic game) is one of my favorite sports.
Show possession with the pronoun “it” by not using the apostrophe.
Answers
10. (A) The three farmers’ fields encompassed over 500 acres.
Three farmers collectively own the fields, so show possession by 1. (A) The large dog eats quite a bit of food each day (mainly food from a
putting an apostrophe after the s. bag).
Put the period outside of the parentheses since the parentheses will not
Parentheses affect the surrounding punctuation.

Parentheses () do not affect the grammar and meaning of the 2. (B) When you eat breakfast (considered the most important meal of the
surrounding sentence. day), you have energy for several hours.
Have the comma outside of the parentheses.
Without Parentheses With Parentheses
Once I finish the test, I will read my book for Once I finish the test (it should take about 30
3. (B) Cricket (once an Olympic game) is one of my favorite sports.
fun. minutes), I will read my book for fun. There is no need for a comma within the parentheses since the
Your next-door neighbor is really nice. Your next-door neighbor (the one with the parentheses already provide a needed pause.
unusual mailbox) is really nice.
My teacher gave everyone the opportunity to My teacher gave everyone the opportunity to Question Marks
earn extra credit after the difficult test. earn extra credit after the difficult test (only a
handful of students took advantage of it). Do use a question mark (?) at the end of a sentence that directly asks a
question.
Why did you bring that toy to school with you? Punctuation Practice Questions
What is he planning on doing in the future?
“Who is knocking at the door?” my father asked. Exercise 1
1. I recently moved to a new building, and I am not so sure about some of
Don’t use a question mark if the sentence indirectly asks a question.
the people who live there. ____________________________ quite a
I would like to find out who my teacher will be. bit of noise when he walks around.
He is excited to find out what happens next in the play.
The article addresses the central question of whether nature or nurture Which choice completes the text so that it conforms to the conventions
ultimately has a greater impact. of Standard English?

Question Mark Practice—select the better sentence out of the two options. (A) The man who lives in the apartment above me makes
(B) The man, who lives in the apartment above me makes
1. (A) If you want to do well in class, be sure to ask your teacher
(C) The man, who lives in the apartment above me, makes
questions?
(D) The man who lives in the apartment, above me makes
(B) If you want to do well in class, be sure to ask your teacher
questions. 2. I spoke to my teacher, and she gave me some good advice. “To do well
2. (A) Where is the nearest subway station. on the SAT, be sure to work on improving several key
(B) Where is the nearest subway station? __________________________________ reasoning, and essay
editing.”
3. (A) While many students are quiet in class, I prefer to ask my teacher
what I need to know. Which choice completes the text so that it conforms to the conventions
(B) While many students are quiet in class, I prefer to ask my teacher of Standard English?
what I need to know?
(A) skills, reading comprehension, quantitative
Answers (B) skills: reading comprehension quantitative
(C) skills—reading comprehension, quantitative
1. (B) If you want to do well in class, be sure to ask your teacher (D) skills; reading comprehension, quantitative
questions.
This is an indirect question, so no question mark is needed. 3. The weather is lovely—there is a gentle breeze, mild temperature, and
no precipitation. _____ a great afternoon to read a book for pleasure.
2. (B) Where is the nearest subway station?
This is a direct question, so use a question mark. Which choice completes the text so that it conforms to the conventions
of Standard English?
3. (A) While many students are quiet in class, I prefer to ask my teacher
what I need to know. This is an indirect question, so no question mark (A) Its
is needed. (B) It’s
(C) Its’ (C) life (sunshine, fresh air, and starlit nights—are
(D) It (D) life—sunshine, fresh air, and starlit nights, are

4. The area of the park for adults had a botanical garden, a koi pond, and 7. When you go shopping, please buy sugar, flour, and butter at the grocery;
several artistic rock formations. The _______ playground, on the other antacids, bandages, and a thermometer at the ___________
hand, had some enormous slides and swings. screwdriver, and some sandpaper from the hardware store. Please
come back with whatever change remains.
Which choice completes the text so that it conforms to the conventions
of Standard English? Which choice completes the text so that it conforms to the conventions
of Standard English?
(A) children’s
(B) childrens’ (A) pharmacy, and a hammer, a
(C) childrens (B) pharmacy: and a hammer a
(D) childs’ (C) pharmacy, and a hammer a
(D) pharmacy; and a hammer, a
5. My history teacher brings history to ____________ activities, guest
speakers, and discussions about controversial topics. It is truly exciting 8. As the human resources officer was immersed in making changes to the
to be a part of her class. employee ________ neglected to enforce the policies already on the
books. Ultimately, he was not very effective in his job.
Which choice completes the text so that it conforms to the conventions
of Standard English? Which choice completes the text so that it conforms to the conventions
of Standard English?
(A) life with stimulating,
(B) life with: stimulating (A) manual, he
(C) life with; stimulating (B) manual he
(D) life with stimulating (C) manual. He
(D) manual; he
6. I came to an important realization as I worried about saving enough
money for the future. Some of the best things in 9. While surfing the Internet, John found that _______________ been
___________________________ completely free. found in a nearby city. He was overjoyed to know that his companion of
many years was safe and sound.
Which choice completes the text so that it conforms to the conventions
of Standard English? Which choice completes the text so that it conforms to the conventions
of Standard English?
(A) life, sunshine, fresh air, and starlit nights are
(B) life—sunshine, fresh air, and starlit nights—are (A) Sprinkles his missing pet cat, had
(B) Sprinkles, his missing pet cat, had would not correctly demonstrate possession, and (D) uses the incorrect
(C) Sprinkles, his missing pet cat had plural version of “child.”
(D) Sprinkles his missing pet cat had
5. (D) The other options provide unnecessary breaks, while choice (D)
10. Mitchell wrote an excellent ________ had it published in a journal. His allows the phrase to properly flow.
family members were thrilled that the talents he had worked so long to
fine-tune were at last being recognized by the public. 6. (B) When setting aside a parenthetical phrase, it is necessary to use the
same punctuation to start and finish the parenthetical phrase. Choices
Which choice completes the text so that it conforms to the conventions (C) and (D) mix and match the beginning and ending punctuation, and
of Standard English? choice (A) does not have punctuation after the end of the parenthetical
phrase. Only choice (B) begins and ends the parenthetical phrase with
(A) poem; he consistent punctuation.
(B) poem, he
(C) poem he 7. (D) This option correctly uses a semicolon to separate the three sets of
(D) poem, he, listed items, each of which has commas within the list. The semicolon
is used in this way to help avoid confusion about the separation of
Answer Explanations listed items, which would result if commas were used in this situation.

1. (A) It is not just any man but specifically the man who lives in the 8. (A) This option correctly separates the dependent introductory clause
apartment above the narrator who makes noise. Since this description from the independent clause with a comma. Choice (B) has no
is essential to identifying the subject of the second sentence, there is no separation. Choices (C) and (D) would have to have a complete
need for any commas to break this up. sentence before the punctuation to be correct.

2. (C) The dash provides a heavy pause before the list of the skills, each 9. (B) This option puts the clarification of the identity of “Sprinkles” off
of which is separated by a comma. Choice (A) does not have a strong to the side by surrounding it with commas. The other options do not
enough punctuation, like a colon or dash, before the list. Choice (B) separate this phrase from the rest of the sentence.
does not have a comma separating the first and second items in the list.
10. (A) The semicolon separates the two independent clauses from one
Choice (D) incorrectly uses a semicolon because what follows the
another. A comma is insufficient when used without a conjunction to
semicolon is not a complete sentence.
separate two independent clauses.
3. (B) “It’s” the contraction for “it is,” making it the correct choice. “Its”
shows possession, “its’” is always incorrect, and “it” does not have Exercise 2
the needed verb. 1. The interconnectedness between mental and physical health has been
consistently shown in a variety of fields in research. _________ causes
4. (A) With a word like “children” that is already plural, you can show
relatively minor physical ailments like headaches, digestive issues, and
possession by adding an apostrophe and an “s.” Choices (B) and (C)
muscle tension.
Which choice completes the text so that it conforms to the conventions 4. With health care professionals more needed than ever, hospitals and
of Standard English? universities are working to attract students to their health care
programs. Students often head into either a nursing or premed track
(A) In the short term mental stress, typically without _____________ there are many more jobs in healing beyond
(B) In the short term, mental stress typically (or better yet, between) those two positions.
(C) In the short term; mental stress typically
(D) In the short term, mental stress, typically Which choice completes the text so that it conforms to the conventions
of Standard English?
2. The most remarkable fact in connection with the season was the earlier
_________ greatly delayed arrival. This was probably due almost (A) realizing however that
entirely to the unusual weather conditions that seemed to prevail (B) realizing however, that
throughout the South during March and early April, affecting the (C) realizing, however, that
migration of these birds. (D) realizing; however that

Which choice completes the text so that it conforms to the conventions 5. Whether they like it or dislike it, most Americans are familiar with the
of Standard English? concept of a _____________ in to save a large company that is
floundering. The government does this primarily to try to save the
(A) migrant’s, economy from potentially crushing bankruptcies of giant corporations
(B) migrant’s, that employ thousands of people.
(C) migrant
(D) migrants’ Which choice completes the text so that it conforms to the conventions
of Standard English?
3. A parasitic relationship is between two __________ of the organisms
benefits while the other is harmed. Parasitic relationships are often (A) government bailout: the government swoops
associated with internal parasites like worms or some species of mites, (B) government bailout, the government swoops
but parasitic relationships are not limited to organisms residing within (C) government; bailout the government swoops
a host. (D) government bailout the government—swoops

Which choice completes the text so that it conforms to the conventions 6. Good health care practitioners should be able to find work in nearly any
of Standard English? corner of the United _______________ pay in less desirable regions
like rural areas and other underserved communities.
(A) organisms, one
(B) organisms—one Which choice completes the text so that it conforms to the conventions
(C) organisms, while one of Standard English?
(D) organisms. If one
(A) States, moreover, they can earn higher
(B) States: moreover they can earn higher (A) Its’
(C) States, moreover they can earn higher (B) Its
(D) States; moreover, they can earn higher (C) It’s
(D) It is
7. The amount of time spent in making observations during the past
_____________ Observations were typically made independently by 10. Like the mental benefits, and thus physical benefits, that have been found
each of the writers and on lands differing somewhat in general from living in close-knit ____________ are many activities that are
character. known to promote physical health due to their impact on mental health.

Which choice completes the text so that it conforms to the conventions Which choice completes the text so that it conforms to the conventions
of Standard English? of Standard English?

(A) season, is significant. (A) communities, there


(B) season is significant. (B) communities there
(C) season is, significant. (C) communities; there
(D) season—is significant. (D) communities: there

8. The migration of birds at Raleigh, N.C., during the spring of 1915 was Answer Explanations
so unusual that it is believed that a short ____________ be of interest
to the readers of Bird-Lore. 1. (B) This option correctly places a comma after the introductory phrase,
separating the dependent clause from the independent clause that
Which choice completes the text so that it conforms to the conventions follows. Choice (A) would interrupt the independent clause, separating
of Standard English? its subject, “mental stress,” from the remainder of the sentence. Choice
(C) incorrectly uses a semicolon after a dependent clause—a
(A) account, together with a list of the records, will semicolon must have an independent clause on either side of it. Choice
(B) account, together with a list of the records will (D) has too many commas.
(C) account—together with a list of the records, will
(D) account together with a list of the records; will 2. (D) Based on the context in the second sentence, there are multiple
birds that are migrating. So, show possession of the “arrival” by putting
9. Upon hatching, a baby European cuckoo’s first instinct is to empty the the apostrophe after “migrants.” It is not choice (A) or (B) since there
nest. _______ back is perfectly shaped to scoop up other eggs and eject are multiple migrants, and it is not choice (C) because this does not
them from the nest. show possession.

Which choice completes the text so that it conforms to the conventions 3. (B) Use a dash before the clarification of the relationship between the
of Standard English? two organisms since it provides a heavy pause. Choice (A) is a comma
splice. Choice (C) incorrectly uses “while” to show a contrast. Choice 10. (A) A comma will correctly separate the long dependent clause from
(D) would result in a sentence fragment for the second part. the independent clause that follows. Choice (B) provides no pause, and
choices (C) and (D) would need a complete sentence preceding their
4. (C) The transitional word “however” should be surrounded with punctuation.
commas since the sentence would still be logical if this word were
removed. Choices (A) and (B) do not have enough commas, and choice Sentence Structure and Organization
(D) incorrectly uses a semicolon since there is not a complete sentence
after the semicolon. Moreover, there is no comma after “however” in Parallelism
choice (D).
Even though the meaning of a sentence may be clear, unless it is written in a
5. (A) The colon is appropriate in this case because there is a complete way that has stylistic consistency, it will not flow how it should. Have
sentence before it and a clarification after it. Choice (B) would result stylistic consistency by looking out for parallel structure.
in a comma splice. Choice (C) would not have a complete and logical
sentence after the semicolon, and choice (D) has a pause at an awkward Example 1
spot.
Incorrect: Two of my hobbies are riding bicycles and to watch movies.
6. (D) When joining two sentences with a conjunctive adverb like
“moreover,” you need a semicolon before the adverb and a comma Correct: Two of my hobbies are riding bicycles and watching movies.
after. Choice (D) is the only option that joins these sentences in this
way. Explanation: Maintain the same way of describing the hobbies so that the
sentence has a parallel structure.
7. (B) Do not separate the subject “the amount of time spent in making
observations during the past season” from the verb “is.” All the other Example 2
options give an unnecessary pause.
Incorrect: Covered in mud, leaves, and in grass, he really needed a shower.
8. (A) Surround the parenthetical phrase, “together with a list of the
records,” with commas. Choice (B) only begins the parenthetical Correct: Covered in mud, leaves, and grass, he really needed a shower.
phrase with a comma, but does not have one at the end. Choice (C) is
inconsistent in how it starts and finishes the parenthetical punctuation. Explanation: If all the items in the list started with “in,” then it would be fine
And choice (D) incorrectly uses a semicolon when no complete to maintain that phrasing. Since the “in” that goes with “mud” also goes with
sentence follows it. “leaves,” it can also apply to “grass” so there is no need to have “in” again.

9. (B) “Its” is the correct form of the possessive for the bird’s back. Parallelism Practice—select the better sentence out of the two options.
Choice (A) would never be correct, and choices (C) and (D) would
1. (A) Choose a teacher to write your letter of recommendation who
signify “it is” instead of the possessive.
knows you well, likes the work you have done, and has the time to do a
good job.
(B) Choose a teacher to write your letter of recommendation who 9. (A) To clean his apartment, Henry vacuumed the floor and dusting the
knows you well, likes the work you have done, and time to do a good shelves.
job. (B) To clean his apartment, Henry vacuumed the floor and dusted the
shelves.
2. (A) To win or losing is not really important—just enjoy the game.
(B) Winning or losing is not really important—just enjoy the game. 10. (A) My brother, a football fan, and my sister, a baseball fan, both enjoy
watching sports in autumn.
3. (A) Neither the children nor the adults were ready to leave the (B) My brother, a football fan, and my sister, who is a baseball fan, both
amusement park. enjoy watching sports in autumn.
(B) Neither the children or those who were adults were ready to leave
the amusement park. Answers
4. (A) To get the most out of class, listen carefully, write detailed notes,
1. (A) Choose a teacher to write your letter of recommendation who
and ask pointed questions.
knows you well, likes the work you have done, and has the time to do a
(B) To get the most out of class, listen carefully, write detailed notes,
good job.
and asking pointed questions.
Use “has the” with the final item in the list since all the listed items
5. (A) Dressed in tuxedos, ballgowns, and additionally in jewels, the begin with a verb.
celebrities were ready for the awards banquet.
(B) Dressed in tuxedos, ballgowns, and jewels, the celebrities were 2. (B) Winning or losing is not really important—just enjoy the game.
ready for the awards banquet. “Winning” and “losing” use the same phrasing.

6. (A) Having a diversified portfolio, including investments in domestic 3. (A) Neither the children nor the adults were ready to leave the
growth stocks and international items, is a sound financial strategy. amusement park.
(B) Having a diversified portfolio, including investments in domestic “Neither” and “nor” go together, while “either” and “or” go together.
growth stocks and international value stocks, is a sound financial
4. (A) To get the most out of class, listen carefully, write detailed notes,
strategy.
and ask pointed questions.
7. (A) Ben, a country music lover, and Kim, an appreciator of the musical “Asking” is not parallel with the other verbs.
genre of classical, surprisingly found a radio station they both liked.
(B) Ben, a country music lover, and Kim, a classical music lover, 5. (B) Dressed in tuxedos, ballgowns, and jewels, the celebrities were
surprisingly found a radio station they both liked. ready for the awards banquet.
Using “additionally in” interrupts the parallel structure.
8. (A) Be sure to pack these items for the bus trip: delicious snacks,
entertaining movies, and good books. 6. (B) Having a diversified portfolio, including investments in domestic
(B) Be sure to pack these items for the bus trip: delicious snacks, growth stocks and international value stocks, is a sound financial
movies that you find entertaining, and good books. strategy.
“International items” does not match the previous example of “domestic Example 2
growth stocks.”
Incorrect: Once we left the stadium, traffic was directed.
7. (B) Ben, a country music lover, and Kim, a classical music lover,
surprisingly found a radio station they both liked. Correct: Once we left the stadium, the police directed the traffic.
Simply using “lover” is sufficient and maintains the parallel structure.
Explanation: The first sentence is vague as to who or what directed the
8. (A) Be sure to pack these items for the bus trip: delicious snacks, traffic. The second sentence makes it clear that the police did so.
entertaining movies, and good books.
Maintain the parallel listing structure of adjective-noun by saying Example 3
“entertaining movies.”
Incorrect: Your book will not become damaged, covered with a brown paper
9. (B) To clean his apartment, Henry vacuumed the floor and dusted the book cover.
shelves.
“Vacuumed” and “dusted” maintain a parallel style. Correct: Your book, covered with a brown paper book cover, will not
become damaged.
10. (A) My brother, a football fan, and my sister, a baseball fan, both enjoy
watching sports in autumn. Explanation: The description of the book—that it is covered with a brown
Since the first parenthetical in the list does not have “who is,” maintain paper book cover—should come after the book, as the second sentence does.
that style for the next parenthetical.
Modifier Placement Practice—select the better sentence out of the two
Modifier Placement options.

Make sure sentences follow a logical order, with descriptions and subjects in 1. (A) I helped when dinner was over to clear the table.
a clear sequence. (B) When dinner was over, I helped to clear the table.
Be certain that the literal meaning matches up with the intended meaning.
2. (A) The deer encountered a bear eating grass in the meadow.
(B) While eating grass in the meadow, the deer encountered a bear.
Example 1
3. (A) My friend Jordan, an art aficionado, loves to go to the latest
Incorrect: After completing the long hike, a three-hour nap did Xavier take. museum openings.
(B) My friend Jordan loves to go to the latest museum openings, an art
Correct: After completing the long hike, Xavier took a three-hour nap.
aficionado.
Explanation: The first sentence does not name the subject until the very end, 4. (A) Widely embraced as an ethical practice, recycling is required in
making it difficult to follow. The second sentence makes it clear that Xavier many municipalities.
took the hike.
(B) Recycling is required in many municipalities, widely embraced as This makes it clear that the narrator helped with clearing the table after
an ethical practice. dinner was over.

5. (A) The 1960s (a decade of much social upheaval) made my relatives 2. (B) While eating grass in the meadow, the deer encountered a bear.
the people they are today. This makes it clear that it is the deer that is eating grass, not the bear.
(B) The 1960s made my relatives the people (a decade of much social
upheaval) they are today. 3. (A) My friend Jordan, an art aficionado, loves to go to the latest
museum openings.
6. (A) My classmate is very generous when I forget my own, always Jordan, not the museum openings, is the art aficionado (someone who
shares his extra pencils and erasers with me. appreciates art).
(B) My classmate is very generous, since he always shares his extra
pencils and erasers with me when I forget my own. 4. (A) Widely embraced as an ethical practice, recycling is required in
many municipalities.
7. (A) Despite my reservations about going on the trip, I ended up having a The ethical practice is recycling, not municipalities.
wonderful time.
(B) I ended, despite my reservations about going on the trip, up having a 5. (A) The 1960s (a decade of much social upheaval) made my relatives
wonderful time. the people they are today.
The 1960s, not the people, were the decade of much social upheaval.
8. (A) Astrophysicists, a mysterious force in the universe, are excited to
discover the true identity of dark energy. 6. (B) My classmate is very generous, since he always shares his extra
(B) Astrophysicists are excited to discover the true identity of dark pencils and erasers with me when I forget my own.
energy, a mysterious force in the universe. This sentence makes sense, putting the reason the classmate is generous
immediately after stating that he is generous, and making the subject
9. (A) The diameter of a circle goes through the center of the circle, which
“he” clear in the later part of the sentence.
is twice the circle’s radius.
(B) The diameter of a circle, which is twice the circle’s radius, goes 7. (A) Despite my reservations about going on the trip, I ended up having
through the center of the circle. a wonderful time.
10. (A) Even though they were delicious, the habanero peppers were so The other option breaks up the phrase “I ended up,” making the
spicy they hurt my eyes. sentence confusing.
(B) The habanero peppers were so spicy they hurt my eyes, even though
8. (B) Astrophysicists are excited to discover the true identity of dark
they were delicious.
energy, a mysterious force in the universe.
Dark energy, not astrophysicists, can logically be described as a
Answers mysterious force in the universe.
1. (B) When dinner was over, I helped to clear the table. 9. (B) The diameter of a circle, which is twice the circle’s radius, goes
through the center of the circle.
This makes it clear that the diameter is twice the circle’s radius, not 1. (A) The downtown park in my city is better than any park.
that the circle is twice its own radius. (B) The downtown park in my city is better than any other park.

10. (A) Even though they were delicious, the habanero peppers were so 2. (A) Your assignment is better than him.
spicy they hurt my eyes. (B) Your assignment is better than his.
The habanero peppers were delicious, not the narrator’s eyes.
3. (A) The principal at my middle school was tougher than the one at my
elementary school.
Logical Comparisons
(B) The principal at my middle school was tougher than my elementary
When making comparisons, be sure that the things being compared match one school.
another in their type and number. As with other sentence structure and
4. (A) I hope that my job application is better than the other applicants.
organization ideas, be sure that the intended meaning corresponds to the
(B) I hope that my job application is better than the other applicants’.
literal meaning.
5. (A) Nurse practitioners provide many of the same treatments that
Example 1 doctors.
(B) Nurse practitioners provide many of the same treatments that
Incorrect: The salaries at the first business are comparable to the other doctors provide.
business.
6. (A) I was disappointed to learn that students who did less work in the
Correct: The salaries at the first business are comparable to those of the class earned better grades than students who put in more effort.
other business. (B) I was disappointed to learn that students who did less work in the
class earned better grades than students of effort.
Explanation: The salaries, not the businesses, are what are being compared.
Putting “those of” in the second part of the sentence makes this clear. 7. (A) Relative to the collection of my local town’s art museum, that of the
Louvre is far more impressive.
Example 2 (B) Relative to the collection of my local town’s art museum, the
Louvre is far more impressive.
Incorrect: Pollen affects me the same way that ragweed.
8. (A) Algebra is better understood as a study of mathematical principles
Correct: Pollen affects me the same way that ragweed does. than simply studying letters.
(B) Algebra is better understood as a study of mathematical principles
Explanation: The ways that pollen and ragweed affect the narrator are what than simply a study of letters.
are being compared. Without putting “does” at the end, the sentence would
compare “pollen affects” to the noun “ragweed.” 9. (A) A general is typically more experienced than a lieutenant.
(B) A general is typically more experienced than a lieutenant does.
Logical Comparisons Practice—select the better sentence out of the two
options. 10. (A) I find walking far easier than running.
(B) I find walking far easier than the times in which I run. 8. (B) Algebra is better understood as a study of mathematical principles
than simply a study of letters.
Answers “A study of” is consistent with the earlier phrasing, not “studying.”

1. (B) The downtown park in my city is better than any other park. 9. (A) A general is typically more experienced than a lieutenant.
Without the “other,” the sentence would be comparing the downtown The general is described with the adjective “experienced,” and there is
park to itself. no need to use a verb like “does.”

2. (B) Your assignment is better than his. 10. (A) I find walking far easier than running.
Compare the assignment to another assignment, not to a person. This comparison concisely maintains the style, ending in “ing.” The
other option is overly wordy and confusing.
3. (A) The principal at my middle school was tougher than the one at my
elementary school. Compare the principal to another principal, not to a Sentence Structure and Organization Practice Questions
school.
1. Michael Jordan, winner of six World Championships, is ____________.
4. (B) I hope that my job application is better than the other applicants’. It is incredible that Jordan accumulated so many titles despite taking a
Using the apostrophe at the end of “applicants” makes the word a hiatus from basketball for several years to pursue his interest in
substitute for the “applications of other applicants,” giving us a logical baseball.
comparison.
Which choice completes the text so that it conforms to the conventions
5. (B) Nurse practitioners provide many of the same treatments that of Standard English?
doctors provide.
Compare the providing of the nurse practitioners to the providing of the (A) as a basketball player who is incredible that is universally
doctors, not the doctors themselves. regarded.
(B) regarded as a basketball player universally and incredibly.
6. (A) I was disappointed to learn that students who did less work in the (C) universally regarded as an incredible basketball player.
class earned better grades than students who put in more effort. (D) as is universally regarded a basketball player with incredible
Keep the phrasing of “who did less work” consistent by using the verb qualities.
“put in.”
2. The playoff match was not even close—the Tigers walloped the Lions
7. (A) Relative to the collection of my local town’s art museum, that of the by 20 points. Undoubtedly, the coach of the Tigers is better than
Louvre is far more impressive. ____________
“That of” provides a stand-in for the collection of the Louvre, so that
you are comparing a collection of one museum to the collection of the Which choice completes the text so that it conforms to the conventions
other. of Standard English?

(A) the Lions.


(B) the coach of the Lions. (B) and in the classroom,
(C) theirs. (C) and classroom,
(D) the other team. (D) and classroom as well,

3. The restaurant was very serene, with only a handful of diners present. 6. The forecasts throughout the week predicted, high winds, heavy
_____________________ He took advantage of the quiet to think about precipitation, and freezing temperatures. However, the weather ended
his plans for the future. up being fine; it _________ hailed. We were immensely relieved.

Which choice completes the text so that it conforms to the conventions Which choice completes the text so that it conforms to the conventions
of Standard English? of Standard English?

(A) My friend, chewing quietly, pensively enjoyed his dinner. (A) either snowed nor
(B) My friend pensively enjoyed his dinner chewing quietly. (B) either snowed and
(C) My friend enjoyed his chewing quietly dinner pensively. (C) neither snowed and
(D) Chewing quietly, his dinner my friend pensively enjoyed. (D) neither snowed nor

4. In the adventure movie, they ran through the forest, fought fierce 7. Out of everyone in my city, I am _______ at solving crossword puzzles.
predators, and _______ the evil villains. I rated the film five stars for My friend, on the other hand, is widely recognized as the best chess
excitement but only three stars for the dialogue. player in town.

Which choice completes the text so that it conforms to the conventions Which choice completes the text so that it conforms to the conventions
of Standard English? of Standard English?

(A) stop (A) good


(B) stopping (B) better
(C) will stop (C) the best
(D) stopped (D) the goodest

5. In the gym, on the field, __________ Hannah acted like a true champion. 8. Upon receiving the notice of losing his job, ________ He took all of his
Her diligence and strength served as an excellent example for both her belongings with him in a small box so that the next employee could use
classmates and teammates. his cubicle.

Which choice completes the text so that it conforms to the conventions Which choice completes the text so that it conforms to the conventions
of Standard English? of Standard English?

(A) and the classroom, (A) his desk was cleared out by him.
(B) he cleared out his desk. 2. (B) By replacing the underlined portion with “the coach of the Lions,”
(C) the desk cleared. the sentence will now make a logical comparison of a coach to a coach.
(D) the desk was cleared. The other options compare the coach to illogical entities.

9. The brand-new _________ Give the affordable price, I decided to buy 3. (A) Choice (A) is the only option that correctly has “chewing quietly”
the extended warranty and an advanced headset. next to what it should modify—“My friend.” The other options put this
phrase in an awkward spot.
Which choice completes the text so that it conforms to the conventions
of Standard English? 4. (D) “Stopped” is the only option that maintains the parallel structure by
having a past tense verb. It is consistent with the other verbs “ran” and
(A) phone, which cost only $30, was covered under the cell phone “fought.” The other choices do not maintain parallelism.
plan I found online.
(B) phone was covered under the cell phone plan I found online, 5. (B) The other phrases in this sentence start with a preposition like “in”
which cost only $30. or “on.” Choice (B) is the only option that begins in that similar
(C) phone that cost only $30 was under the cell phone plan covered, fashion.
that is, the one I found online.
(D) phone which cost only $30 was online under the cell phone I 6. (D) Based on the preceding sentence and the transition “however,” this
found covered. sentence needs to say that no bad weather happened. “Neither . . . nor”
is the appropriate way to express this idea. With “either” and “neither,”
10. I find that driving the sports car is both exhilarating _________ My use “or” not “and.”
friends enjoy going for rides with me, especially when I retract the roof
7. (C) Since there are surely three or more people in the city, the
and turn the car into a convertible.
superlative “best” is the correct word to express this comparison.
Which choice completes the text so that it conforms to the conventions “Good” is typically used to describe a single object, “better” is used
of Standard English? for comparing two objects, and “goodest” is not a word.

(A) and relaxed. 8. (B) This option puts the sentence in an active voice and has the words
(B) and relaxing. in a logical order. Choices (A) and (D) invert the word order and
(C) or relaxing. choice (C) would not make sense since the desk is not doing the
(D) or relaxed. clearing.

9. (A) The phrase “which cost only $30” is nonessential to the sentence
Answer Explanations and can be surrounded with commas. Moreover, the phrase needs to
modify “phone” since that is the item that has the cost of $30. Choice
1. (C) This option puts the words in the most logical sequence. The other
(A) is the only option that does both tasks.
options put the words in sequences that are not very clear.
10. (B) This maintains parallelism with “exhilarating” by having the same sentences—“Still” works well. When working through transition questions,
ending of “ing.” Also, with the word “both,” you need to use “and” be certain to consider the surrounding context so you can determine the best
instead of “or.” logical relationship.

Transitions In the above example, you needed to look at the sentence before and after the
transitional word. In some sentences, the transition is based just on what
The SAT tests your proficiency with transitional words—words like “but,” comes after the transitional word, like “Although I was excited to watch the
“also,” and “because” that connect phrases and sentences together. An movie, I had difficulty finding time to do so.” Err on the side of considering a
understanding of transitional words will ensure that your writing is well- bit too much context just to be sure you fully understand what logical
organized. Here is a list of some of the most common transitional words you relationship is needed.
will encounter on the SAT:
Example 2
General Meaning Transitional Words
Additional Information Also, and, besides, further, furthermore, in addition, what’s more When school begins in the fall, seniors often ask their past teachers to
Cause and Effect As a result, because, consequently, therefore, thus, to that end
write letters of recommendation on their behalf. The best teachers are
typically inundated with requests, and usually have to limit the number
Clarification At any rate, in other words, in fact
of letters they can write. _________ it is a good idea to ask teachers in
Comparison Likewise, similarly, by the same token
the spring or summer for letters of recommendation so that the teachers
Contrast After all, alternately, although, but, by contrast, even though, however, have adequate time to do them.
instead, meanwhile, nevertheless, nonetheless, on the other hand,
rather, still, whereas
Which choice completes the text with the most logical transition?
Give an Example For example, for instance
Obviously Of course
(A) Also,
Time Sequence Eventually, finally, in the first place, next, ultimately (B) For instance,
Typically Traditionally (C) Therefore,
(D) Furthermore,
Example 1
Explanation: The correct answer to this question is (C), “Therefore.” This is
Incorrect: The kitchen had virtually no food in it. Also, we managed to make the only option that provides a cause-and-effect relationship between the first
dinner. two sentences and the final sentence of the paragraph. A cause-and-effect
relationship is necessary here since it is because good teachers are flooded
Correct: The kitchen had virtually no food in it. Still, we managed to make with letter of recommendation requests that it is sensible to ask teachers
dinner. earlier to write letters on your behalf.
Explanation: Even though the kitchen did not have much food, the group was Transitions Practice—select the better sentence(s) out of the two options.
still able to make dinner. Thus, a contrast is needed between the two
1. (A) Even though I had plenty of sleep, I still felt tired. (B) A perfect score would, of course, be preferable; however, a 99 out
(B) Consequently I had plenty of sleep, I still felt tired. of 100 is truly excellent.

2. (A) A part-time job during high school is a great way to make extra 8. (A) We did not do very well in the game; consequently, our coach made
money. Furthermore, it can help you develop skills that will serve you us run extra laps.
well in college and in your career. (B) We did not do very well in the game; and our coach made us run
(B) A part-time job during high school is a great way to make extra extra laps.
money. In contrast, it can help you develop skills that will serve you
well in college and in your career. 9. (A) The roller coaster had an awesome first hill, and what’s more, it
had three incredible loops.
3. (A) When making bread, first preheat the oven. Therefore, put the well- (B) The roller coaster had an awesome first hill and despite this, it had
kneaded dough into the oven to bake. three incredible loops.
(B) When making bread, first preheat the oven. Next, put the well-
kneaded dough into the oven to bake. 10. (A) The wildlife sanctuary had elephants and rhinos. In contrast, it had
zebra, giraffes, and water buffalo.
4. (A) The yard was completely covered with weeds. Nevertheless, the (B) The wildlife sanctuary had elephants and rhinos. Further, it had
gardener happily tackled the weeding and turned the yard into a zebra, giraffes, and water buffalo.
beautiful site.
(B) The yard was completely covered with weeds. Also, the gardener 11. (A) His anger at his neighbor was understandable. Therefore, it was no
happily tackled the weeding and turned the yard into a beautiful site. justification for vandalism of his neighbor’s house.
(B) His anger at his neighbor was understandable. However, it was no
5. (A) Majoring in two different areas may allow for a natural synergy justification for vandalism of his neighbor’s house.
between subjects. For instance, a business major with an engineering
major could help you learn both how to make a product and how to 12. (A) The world-renowned orchestra conductor is nonetheless an
market it. excellent violinist.
(B) Majoring in two different areas may allow for a natural synergy (B) The world-renowned orchestra conductor is likewise an excellent
between subjects. Whereas a business major with an engineering major violinist.
could help you learn both how to make a product and how to market it.
13. (A) I did not eat breakfast. As a result, I was really hungry for lunch.
6. (A) He did not have a good reason to run for office, and, by the same (B) I did not eat breakfast. Moreover, I was really hungry for lunch.
token, he did not have a reason to avoid running.
14. (A) Thanksgiving is, in other words, a day on which people eat quite a
(B) He did not have a good reason to run for office, and, in the first
bit of food.
place, he did not have a reason to avoid running.
(B) Thanksgiving is traditionally a day on which people eat quite a bit
7. (A) A perfect score would, of course, be preferable; thus, a 99 out of of food.
100 is truly excellent.
15. (A) Her parents did not think she wanted to join the club. In fact, she
was the first one to sign up for the club at the school fair.
(B) Her parents did not think she wanted to join the club. Typically, she “Furthermore” makes sense since the idea of skill development would
was the first one to sign up for the club at the school fair. be an additional benefit of having a part-time job.

16. (A) Best friends are forever; despite this, you will stay connected to 3. (B) When making bread, first preheat the oven. Next, put the well-
your best friend over many decades and great distances. kneaded dough into the oven to bake.
(B) Best friends are forever; in other words, you will stay connected to “Next” demonstrates the sequence of events that one would find in a
your best friend over many decades and great distances. recipe.
17. (A) At first, the athlete resisted the advice of his coach. Ultimately, he 4. (A) The yard was completely covered with weeds. Nevertheless, the
took the coach’s advice and improved immensely. gardener happily tackled the weeding and turned the yard into a
(B) Initially, the athlete resisted the advice of his coach. At first, he took beautiful site.
the coach’s advice and improved immensely. “Nevertheless” shows a contrast between the fact that the yard was
covered with weeds and the fact that the gardener could still make the
18. (A) I am excited for the first day of school. On the other hand, I am
yard into something beautiful.
worried about a pop quiz over the summer reading.
(B) I am excited for the first day of school. Consequently, I am worried 5. (A) Majoring in two different areas may allow for a natural synergy
about a pop quiz over the summer reading. between subjects. For instance, a business major with an engineering
19. (A) We climbed up the mountain and still reached the summit. major could help you learn both how to make a product and how to
(B) We climbed up the mountain and eventually reached the summit. market it.
“For instance” is a logical way to join the two sentences, since the
20. (A) The doctor said you should eat more fruits and vegetables. second sentence provides an example of how a double major could
Therefore, you should come to the farmer’s market to buy some healthy make sense.
produce.
(B) The doctor said you should eat more fruits and vegetables. Whereas 6. (A) He did not have a good reason to run for office, and, by the same
you should come to the farmer’s market to buy some healthy produce. token, he did not have a reason to avoid running.
“By the same token” means that the man both had a good reason to run
Answers for office and had a reason not to run.

7. (B) A perfect score would, of course, be preferable; however, a 99 out


1. (A) Even though I had plenty of sleep, I still felt tired.
of 100 is truly excellent.
“Even though” shows contrast between having plenty of sleep and still
“However” shows a contrast between the acknowledgement that a
feeling tired.
perfect score would be better, but that a 99 is still outstanding.
2. (A) A part-time job during high school is a great way to make extra
8. (A) We did not do very well in the game; consequently, our coach made
money. Furthermore, it can help you develop skills that will serve you
us run extra laps.
well in college and in your career.
“Consequently” states the cause-and-effect relationship between not
doing well in the game and the coach’s decision to make the players run
laps. 16. (B) Best friends are forever; in other words, you will stay connected to
your best friend over many decades and great distances.
9. (A) The roller coaster had an awesome first hill, and what’s more, it The part of the sentence after the semicolon clarifies what is meant by
had three incredible loops. the statement that “best friends are forever,” so “in other words” is
“What’s more” shows that in addition to the awesome first hill, the logical.
roller coast had three incredible loops.
17. (A) At first, the athlete resisted the advice of his coach. Ultimately, he
10. (B) The wildlife sanctuary had elephants and rhinos. Further, it had took the coach’s advice and improved immensely.
zebra, giraffes, and water buffalo. This sentence puts the sequence of events in a logical order, with the
“Further” shows that along with the elephants and rhinos, the sanctuary athlete initially resisting the coach’s advice and eventually following it.
also had other animals.
18. (A) I am excited for the first day of school. On the other hand, I am
11. (B) His anger at his neighbor was understandable. However, it was no worried about a pop quiz over the summer reading.
justification for vandalism of his neighbor’s house. “On the other hand” suggests a contrast between the excitement of the
“However” shows that while the man was understandably angry at his student and the worry over a quiz.
neighbor, he had no good reason to vandalize his neighbor’s home.
19. (B) We climbed up the mountain and eventually reached the summit.
12. (B) The world-renowned orchestra conductor is likewise an excellent This option shows a logical sequence of events with “eventually” used
violinist. to mark the final part of the journey.
“Likewise” shows that the conductor is both excellent with the
orchestra and with the piano. 20. (A) The doctor said you should eat more fruits and vegetables.
Therefore, you should come to the farmer’s market to buy some healthy
13. (A) I did not eat breakfast. As a result, I was really hungry for lunch. produce. “Therefore” shows a cause-and-effect relationship between
“As a result” shows a direct cause-and-effect relationship between not the doctor’s recommendation about how to eat and the action necessary
eating breakfast and being hungry for lunch. to acquire the recommended foods.
14. (B) Thanksgiving is traditionally a day on which people eat quite a bit
Transitions Practice Questions
of food.
“In other words” does not make sense since the sentence is not Exercise 1
rephrasing anything; “traditionally” works because the sentence says
what typically happens on this holiday. 1. The two friends saw eye to eye on virtually everything, from food to
fashion, and from literature to music. When one found something to be
15. (A) Her parents did not think she wanted to join the club. In fact, she funny, the other was ________ amused.
was the first one to sign up for the club at the school fair. “In fact”
clarifies that despite what her parents thought, the daughter was actually Which choice completes the text with the most logical transition?
interested in joining the club.
(A) likewise
(B) specifically 5. The meal looked and smelled a bit strange; _______, it tasted delicious.
(C) finally It is often helpful to be open to new experiences, not being afraid to
(D) meanwhile take that first bite.

2. Electing Maria to be class president was a foregone conclusion. She Which choice completes the text with the most logical transition?
was popular with virtually every group in school. ________ her speech
made a clear and convincing case for her candidacy. (A) therefore
(B) nevertheless
Which choice completes the text with the most logical transition? (C) for this reason
(D) to that end
(A) Notwithstanding,
(B) Immediately, 6. When you want to chop vegetables, begin by grabbing the ingredients
(C) Although, you will need. ______, get the cutting board and knife. Finally, begin
(D) Moreover, chopping the vegetables into the shapes and sizes you need for the dish.

3. The court proceedings were about to begin without the sole eyewitness Which choice completes the text with the most logical transition?
to the crime present. The judge _______ summoned the eyewitness to
appear at the next court hearing to help get to the bottom of the matter. (A) First
(B) At last
Which choice completes the text with the most logical transition? (C) In short
(D) Second
(A) in contrast
(B) still 7. As a father, I try to give my children the best advice I can. ________, I
(C) accordingly am no expert in psychology. However, I genuinely believe that I have
(D) despite this my children’s best interests at heart and want to help however possible.

4. The play will be delayed for several minutes so that the ushers can Which choice completes the text with the most logical transition?
guide the latecomers to their seats. ________ spectators can spend
some time checking out the program and enjoying their concessions. (A) Granted
(B) Similarly
Which choice completes the text with the most logical transition? (C) Beyond
(D) Foundationally
(A) Centrally,
(B) In the meantime, 8. The squirrel grabbed a large walnut and took it up a tree for storage.
(C) To summarize, Only a few minutes ______ the squirrel returned to grab yet another
(D) Furthermore,
walnut. With dedication like this, the squirrel should be in excellent 2. (D) Maria is popular with everyone and in addition her speech was
shape for a long winter. very convincing. The most appropriate transition is “moreover” since it
means “also.” The other options do not convey this relationship.
Which choice completes the text with the most logical transition?
3. (C) “Accordingly” shows a cause-and-effect relationship, which makes
(A) afterward sense given that the sole witness was absent and the judge wanted the
(B) beyond witness to appear. The other options do not show a cause-and-effect
(C) nevertheless relationship.
(D) therefore
4. (B) “In the meantime” means that someone can do something else while
9. The baker wanted to make something that would be acceptable for another thing is happening. The spectators can do other things as they
diners with a gluten intolerance. _________ in mind, he found some wait for the delayed play to begin, making “in the meantime” an
gluten-free flour and made a wonderful dessert. appropriate transition. The other options do not convey this idea.

Which choice completes the text with the most logical transition? 5. (B) “Nevertheless” shows a contrast, unlike the other options. A
contrast is appropriate in this context since although the meal seems
(A) Unfortunate strange, it ended up tasting quite good.
(B) In fact of purpose
(C) Truly 6. (D) The text is outlining the steps that you should take when chopping
(D) With this object vegetables. The second step out of the three steps is to get the cutting
board and knife, making choice (D) the only logical option.
10. When you write your term paper, be sure to give plenty of examples
________ your primary argument. The grader will carefully examine 7. (A) “Granted” is synonymous with “it is true that . . .” In this context,
how strong the support is for your thesis. the narrator Is stating that while he tries to give the children the best
possible advice, he may fall short in the quality of the advice since he
Which choice completes the text with the most logical transition? is not an expert in psychology. The other options do not acknowledge
the truth of the situation.
(A) with contend
(B) of claiming 8. (A) The squirrel returned “afterward” to get another walnut after
(C) on advance initially grabbing and storing one. The other options would not convey
(D) to illustrate the timeline of events.

Answer Explanations 9. (D) “With this object” means “with a goal.” In this context, the baker
wants to make something for diners who cannot eat gluten; given this
1. (A) The two friends are similar in all their beliefs, so the transition goal, he finds flour that allows him to make something they can enjoy.
“likewise” expresses this similarity. The other options do not express the appropriate connection.
10. (D) The grader is looking for support for the thesis, so it is important not have the power to intervene in a meaningful way. ______ Morgan
“to illustrate” your argument. The other options are grammatically did.
incorrect.
Which choice completes the text with the most logical transition?
Exercise 2
(A) So,
1. The parasitic European cuckoo decreases the mother bird’s output and, (B) Additionally,
in some cases, kills all offspring, ________ decreasing the host bird’s (C) Formerly,
fitness. On top of reproductive output, the mother bird must put in (D) Moreover,
additional work to feed the equivalent of a nest full of babies that is just
one large baby. 4. The Roseto Effect was first studied in Roseto, Pennsylvania, where
there lived a tight-knit community whose diets contained significant
Which choice completes the text with the most logical transition? levels of fatty acids. _________ members of the town eating high
levels of fats, drinking alcohol, and commonly smoking tobacco, the
(A) however town’s residents had significantly lower rates of cardiovascular
(B) but diseases than those in surrounding areas.
(C) therefore
(D) previously Which choice completes the text with the most logical transition?

2. “Ladies and gentlemen of the class of ’97, wear sunscreen. If I could (A) Despite
only offer you one tip for the future, that would be it.” ______ begins a (B) Because of the
well-known commencement speech; the speaker goes on to explain why (C) Given the
his advice about careers and life goals may or may not be helpful to the (D) Whenever
students, but the advice to wear sunscreen will always be applicable.
5. While the concept of a government bailout is fairly new, the idea of
Which choice completes the text with the most logical transition? external powers stepping in to try to boost the economy is not. Prior to
the era of governmental intervention, _________ it was big business
(A) Whereas who worked to protect the economy and with it, their profits.
(B) Additionally
(C) Thus Which choice completes the text with the most logical transition?
(D) Also
(A) consequently,
3. In 1907 when a financial crisis (one of the foreshocks of the panic that (B) also,
would lead to the Great Depression) took place, Morgan took charge. (C) due to this,
Banks were on the brink of falling apart. People were going to lose (D) however,
their savings; the situation was dire and the government at the time did
6. Easily one of the most distinctive painters associated with the surrealist (D) but
movement of the early 20th century, Frida Kahlo _________ insisted
adamantly throughout her career that she was by no means a surrealist 9. Sunlight is different from the light that a person experiences from a fire
herself. or a light bulb. Sunlight is mostly UV light, which has a much shorter
wavelength than the light we turn on when we use the bathroom in the
Which choice completes the text with the most logical transition? middle of the night. __________ if the earth’s protective ozone layer
didn’t filter out much of the UV light, humans would risk their health
(A) consequently just by going about their everyday lives during daylight hours.
(B) moreover
(C) accordingly Which choice completes the text with the most logical transition?
(D) nonetheless
(A) Meanwhile,
7. A good broad range sunscreen can, when used properly, prevent sunburn (B) Nonetheless,
for hours. __________ if a product has a sun protection factor (SPF) of (C) Indeed,
30, that means that skin can be exposed for 30 times longer without (D) As a result,
burning than if no protection was used.
10. If school were operated on a year-round basis, student retention of
Which choice completes the text with the most logical transition? material would be strengthened since it is much easier to forget
material over a three-month break than over a two-week one. Material
(A) Despite this, retention is especially vital for subjects like math and foreign language
(B) For example, in which more advanced units will not make sense without a sound
(C) In contrast, basis in the earlier material. __________ if students have more
(D) Also, frequent breaks from the rigors of school, they are less likely to burn
out.
8. Despite its sinister and somewhat draconian reputation,
electroconvulsive therapy (ECT) is making a rather surprising return in Which choice completes the text with the most logical transition?
the treatment of chronic clinical depression. Like many of its
pharmaceutical counterparts, the exact physiological mechanism (A) However,
responsible for ECT’s alleviation of the symptoms of depression (B) Consequently,
remains elusive, ______ its results are inarguably remarkable. (C) Additionally,
(D) On the other hand,
Which choice completes the text with the most logical transition?
Answer Explanations
(A) and
(B) since 1. (C) This option is the only choice that shows a cause-and-effect
(C) with relationship between the European cuckoo acting parasitically toward
other birds and the resulting decrease in the host bird’s overall fitness. 7. (B) “For example” is appropriate since the current sentence provides
Choices (A) and (B) show a contrast, and choice (D) unnecessarily an example of what makes a high-quality sunscreen. It is not choices
inserts a time sequence indication. (A) or (C) because there is not a contrast, and it is not choice (D)
because the current sentence provides an example, not simply an extra
2. (C) “Thus” is used to make the idiomatic phrase “Thus begins”; this point.
phrase provides a logical transition between the quotation that started
the speech and the clarification of the significance of the speech. It is 8. (D) The word “but” is the appropriate transition. The sentence contains
not choice (A) because there is no contrast, and it is not choices (B) or a contrast—scientists do not know exactly how ECT works, but they do
(D) because this is not an additional piece of information, but a know that the results are remarkable. The transitions in choices (A),
clarification. (B), and (C) do not indicate a contrast.

3. (A) “So” correctly expresses a cause-and-effect relationship between 9. (C) “Indeed” is used to introduce a further point that elaborates on the
the earlier sentence, which says that there was a dire situation with no previous point made; in this case, “indeed” connects the previous
government help, and the current sentence expresses that Morgan did sentence that states that sunlight is mostly UV light and the current
intervene. The other options do not express a cause-and-effect sentence that underscores the potential danger from UV light to humans.
relationship. It is not choices (A) or (B) because there is not a contrast, and it is not
choice (D) because there is not a cause-and-effect relationship.
4. (A) “Despite” signifies an oppositional relationship between the first
part of the sentence, which states that the diets of the town’s residents 10. (C) The word “Additionally” correctly indicates that this sentence
are not particularly healthy, and the unexpected lack of cardiovascular builds upon the argument in the previous sentences. Choice (B),
disease among the residents. The other options do not convey a “Consequently,” indicates a cause-and-effect relationship. Choice (A),
contrasting relationship. “However,” and choice (D), “On the other hand,” both indicate a
contrast.
5. (D) “However” is the appropriate transition in this context, since the
sentence points out the contrast between the current era of governmental Rhetorical Synthesis Questions
intervention and an era in the past when privately based bailouts were
more commonplace. Choices (A) and (C) would show cause and effect, A new type of question on the Digital SAT is the rhetorical synthesis, or
and choice (B) would be used to list an additional item or idea. “rhetorical analysis,” question. You will be asked to consider the notes that a
student has taken on a topic and determine the best way to use the information
6. (D) “Nonetheless” is used to show a contrast between the fact that in the notes to present or emphasize a particular idea. On questions like
Frida Kahlo is considered a surrealist by society, yet she did not these, be sure to really focus on what the question asks so that you can zero in
consider herself a surrealist. Choice (A), “consequently,” and choice on the most relevant evidence.
(C), “accordingly,” show cause and effect. Choice (B), “moreover,” is
synonymous with “also.” This is a good example where you can use the Example
similarities among choices (A) and (C) to eliminate them.
While researching a topic, a student has taken the following notes:
The classical Humanist ideal of human individualism can be represented by extensive compassion, and Posthumanism is in fact open to the consideration
Leonardo da Vinci’s “Vitruvian Man.” of the needs of nonhumans.
The Vitruvian Man was an emblem of Humanism and the source for the
civilization model that assumes Europe is not just a location, but a universal Rhetorical Synthesis Practice Questions
standard of humankind.
Posthumanism tends to turn away from the nature and culture binary and Exercise 1
questions the distinctions between what is considered human and nonhuman. Select the best choice out of the four options.
Posthumanism rejects individualism by discouraging human-centered, or
self-centered, thought, and instead urges interconnection between self, 1. While researching a topic, a student has taken the following notes:
others, and nonhuman others.
Personal trainers are fitness experts who help clients implement their health
The student wants to emphasize what makes Humanism different from and exercise goals.
Posthumanism. Which choice most effectively uses relevant information from Trainers often personally guide their clients with customized workouts.
the notes to accomplish this goal? They can help clients with their greatest areas of need, like endurance,
strength, or flexibility.
(A) Humanism focuses on a concern for others, while Posthumanism Trainers ensure that clients train safely, helping them maintain the proper
embraces self-centeredness. exercise form.
(B) Humanism argues for individualism, while Posthumanism rejects Some trainers choose to pursue a four-year degree in exercise science or a
it. related field, while others complete a one-year certificate program.
(C) Humanism focuses on geography, while Posthumanism is In addition to a formal education, trainers are almost always expected to
concerned with politics. have CPR training.
(D) Humanism advocates compassion for all living things, while
Posthumanism focuses on the needs of humans alone. The student wants to present a how personal trainers can prevent
clients from becoming injured. Which choice most effectively uses
Answers relevant information from the notes to accomplish this goal?

The correct answer is choice (B). Focus on the specific question: “The (A) Personal trainers often pursue educational credentials, like an
student wants to emphasize what makes Humanism different from exercise science degree, that gives their clients confidence in
Posthumanism.” Look at the bullets to determine what makes the two ideas their skills.
different. The first bullet shows that individualism is associated with (B) For clients who need help with building their strength and
classical Humanism. The final bullet shows that Posthumanism rejects endurance, a personal trainer is an excellent option.
individualism. So, choice (B) is correct in articulating the differences (C) Personal trainers can customize workouts for clients, improving
between these two ideas. It is not choice (A) because this gets the two ideas their chances of achieving their fitness goals.
backwards. It is not choice (C) because Humanism is not concerned with (D) Through monitoring the proper exercise form of their clients,
geography whatsoever, and Posthumanism is only loosely related to politics. personal trainers help clients train safely.
It is not choice (D) because Humanism focuses on individualism rather than
2. While researching a topic, a student has taken the following notes: Today, the Met continues to perform at Lincoln Center in New York City
More than 70 percent of the Earth’s surface is covered with water. The student wants to present a likely emotional cause for the formation
Water makes up most of the human body. of the Met. Which choice most effectively uses relevant information
Water is a rather simple compound: 2 hydrogen atoms covalently bonded to from the notes to accomplish this goal?
1 oxygen atom.
Water has several unique chemical properties that make it rather suitable (A) Feelings of exclusion from the elite society of the Academy of
for life. Music likely motivated wealthy families to create an alternative
Water is considered a universal solvent, enabling cells to use nutrients and organization.
oxygen. (B) A pure love of musical artistry by wealthy families most likely
propelled them to initially create the Academy of Music.
The student wants to emphasize the widespread prevalence of water. (C) It is remarkable that the Met has endured over the span of three
Which choice most effectively uses relevant information from the notes different centuries.
to accomplish this goal? (D) After the closure of the Academy of Music, wealthy families who
earnestly missed operatic performances created the Met.
(A) Covering over 70 percent of the Earth’s surface, water is widely
found on our planet.
(B) Since it is a universal solvent, water is quite useful in biological 4. While researching a topic, a student has taken the following notes:
processes. Claire of the Sea Light is a novel by Edwidge Danticat.
(C) The simplicity of the composition of water belies the complexity In the work, the woes of Haiti are addressed directly and indirectly.
of the organisms in which it is found. The narrative threads through the lives of a town where everybody knows
(D) Many chemical compounds, including water, make up the human everybody and no one tragedy can exist on its own.
body. The novel emphasizes several themes:
displacement and exile
3. While researching a topic, a student has taken the following notes: disrupted familial relationships,
the inheritance of trauma and loss
During the late 1800s, the private boxes at the Academy of Music Opera
the fluidity of personal identity and belonging
House were reserved for the most elite families in New York City.
The novel’s attention to current sociopolitical issues offers a more
After being excluded from the Academy of Music private boxes, some
multifaceted and activist-oriented personal account than some of Danticat’s
newly wealthy families met to create a new opera house.
other novels.
The new opera house would be more extravagant than the Academy of
Music one, giving the families an opportunity to display their great wealth. The student wants to summarize how Danticat treats the concept of
The newly formed opera house was called the Metropolitan Opera house, personal identity in Claire of the Sea Light. Which choice most
known today as the “Met.” effectively uses relevant information from the notes to accomplish this
Just three years after the formation of the Met, the Academy of Music shut goal?
down as people flocked to the new performing center.
(A) Danticat articulates how family is at the core of what gives people Answer Explanations
their authentic identity.
(B) Danticat shows that tragedy can affect not just one person but 1. (D) The question asks us to present how personal trainers can prevent
everyone in a town. client injury. Choice (D) accomplishes this task by stating that through
(C) Danticat observes how activists tend to experience a reduction in making sure clients use the proper exercise form, the clients will avoid
social exile. injury. Choice (A) focuses on education instead of injury prevention.
(D) Danticat emphasizes how personal identity can be ever-changing Choice (B) focuses on strength and endurance, which is not nearly as
instead of constant. directly related to safe training as choice (D). Choice (C) vaguely
mentions client workout customization.
5. While researching a topic, a student has taken the following notes:
2. (A) “Prevalence” means commonality—choice (A) shows just how
An adjuvant is a substance that enhances the body’s immune response to an widespread and common water is on our planet. It is not choice (B)
antigen. because this is much more indirect in suggesting the prevalence of
Adjuvants are used in the field of medicine. water. It is not choice (C) because this focuses on complexity instead of
They are used as a possible solution to antibiotic resistant bacteria in commonality. It is not choice (D) because this limits the discussion to
patients. the human body instead of discussing how widespread water is.
Gaston Ramon, a French veterinarian, discovered adjuvants while
observing and treating diseases in horses. 3. (A) The notes state that “after being excluded from the Academy of
Adjuvants are coupled with antibiotics to re-sensitize bacteria to Music private boxes, some newly wealthy families met to create a new
antibiotics, making the antibiotics effective again. opera house.” Based on this, the likely emotional cause for the
E. coli without the transcription factor, EF-P, are more sensitive to formation of the Met was a feeling of exclusion, as noted in choice (A).
antibiotics than E. coli with it. It is not choice (B) because there is no evidence that a pure love of
music was the primary motivation. It is not choice (C) because this has
The student wants to emphasize how modern doctors might use nothing to do with an emotional cause. It is not choice (C) because the
adjuvants to improve patient outcomes. Which choice most effectively Met was created before the end of the Academy of Music, helping to
uses relevant information from the notes to accomplish this goal? bring about its demise.

(A) Doctors might use adjuvants to help patients who are infected 4. (D) The question asks us to summarize how Danticat treats the concept
with antibiotic-resistant bacteria. of personal identity in the novel. Choice (D) effectively accomplishes
(B) Adjuvants can be especially useful in helping those in emergency this task by paraphrasing the note that a theme of the novel is “the
room settings. fluidity of personal identity and belonging.” Choice (A) is incorrect
(C) An adjuvant is the most helpful treatment for patients with an E. because Danticat highlights “disrupted familial relationships,” not how
coli infection. family is the key to personal identity. Choices (B) and (C) do not relate
(D) Veterinarians discovered the extremely useful technology of to personal identity.
adjuvants; now the technology is used on human patients.
5. (A) The notes state that adjuvants are used “as a possible solution to (D) Mitosis and mitosis both allow for exact copies of an organism to
antibiotic resistant bacteria in patients.” Therefore, if a student wants to be replicated generation after generation.
show how the adjuvants can be used to improve patient outcomes,
emphasizing how they reduce infection in antibiotic resistant bacteria 2. In preparing for a physics experiment on simple harmonic motion
makes sense. Choice (B) is not supported by the notes. While E. coli (SHM), a student has taken the following notes:
are mentioned in the final bullet, the point is not on whether adjuvants
SHM is periodic motion where the restoring force is proportional to the
are particularly helpful but on the type of E. coli that is more sensitive
displacement of the object in motion.
to antibiotics—this makes choice (C) incorrect. Choice (D) does not
Examples of SHM are the simple pendulum and an object on a spring.
elaborate on the relative effectiveness of adjuvants.
Periodic motion involves the oscillation of an object about an equilibrium
or lowest potential energy position.
Exercise 2 Formulas for SHM assume relatively small oscillations.
Select the best choice out of the four options. The experiment will evaluate the impact of changing the mass of the
attached object and the length of the string on the length of the oscillations
1. While researching a topic, a student has taken the following notes:
The student wants to emphasize the type of intervals at which an object
In single-celled organisms, mitosis is used for reproduction. in simple harmonic motion will most likely oscillate. Which choice
Mitosis also occurs in more complex organisms for tissue growth and most effectively uses relevant information from the notes to accomplish
replacement of cells. this goal?
In mitosis, the nucleus splits to form two sets of identical chromosomes
resulting in two daughter cells with the same genetic material. (A) When moving in a simple harmonic, an object will likely have
Meiosis is cell division particular to organisms that use sexual oscillations at irregular intervals.
reproduction. (B) Objects in simple harmonic motion often utilize an object on a
Meiosis splits the number of chromosomes in parent cells in half so that spring.
they can combine in offspring. (C) Experiments are needed to determine more about the mass of
Meiosis is essential for genetic diversity. attached objects in a simple harmonic motion.
(D) Objects in simple harmonic motion most likely move in a regular
The student wants to emphasize a significant similarity between mitosis pattern.
and meiosis. Which choice most effectively uses relevant information
from the notes to accomplish this goal? 3. While researching a topic, a student has taken the following notes:
(A) Mitosis and meiosis are both used by living things for The bullroarer is an instrument made of a wooden slat typically measuring
reproduction. between 6 and 24 inches long, affixed at one end to a length of twisted cord.
(B) Mitosis maintains the same genetic material, while meiosis leads The bullroarer is widespread among ancient cultures, and is also known as
to greater diversity. a “tundun,” or “whizzing-stick.”
(C) Mitosis and meiosis are used equally by organisms throughout the The oldest known bullroarers were discovered in the Ukraine, and are
Earth. estimated to date from the Paleolithic era, approximately 17,000 B.C.E.
The sound of the bullroarer is rather unique and has been likened both to an (A) Arrested Justice: Black Women, Violence, and America’s Prison
animal’s roar and the approach of a distant thunderstorm. Nation is a work by the noted feminist Beth Richie.
Pitch modulation can be achieved by altering the speed of rotation, or the (B) Richie epitomizes the feminist quest to understand social
length of the cord. inequality by endeavoring to understand why black women have
been left in vulnerable positions.
The student wants to emphasize the long duration of the bullroarer in (C) Feminism examines why social inequality exists so that there can
human history. Which choice most effectively uses relevant information be a more equitable society.
from the notes to accomplish this goal? (D) The inequality of incarceration is something that many
contemporary authors have examined.
(A) With a sound like a thunderstorm or animal roar, it is no wonder
that the bullroarer has been popular for quite a while. 5. While researching a topic, a student has taken the following notes:
(B) While Ukraine is the location of a bullroarer discovery, other
locations around the world can make a similar claim to fame. A spectrometer is an instrument that spreads a wave of electromagnetic
(C) Having been by humans since at least the Paleolithic era, the radiation into its component frequencies.
bullroarer has been a part of human music for at least 17 If you look at a simple beam of white light through a spectrometer, you will
millennia. see a continuous band of colors shifting like a rainbow from red to violet.
(D) More ancient than any other instrument in human history, the When a material is heated, you can look at the flames given off through a
bullroarer is a foundational component of many civilizations. spectrometer to determine what chemicals make up the material.
This is possible because the band of light is broken into a series of lines
4. While researching a topic, a student has taken the following notes: that represent specific frequencies of electromagnetic radiation that are
emitted from the compound.
Feminism endeavors to create equality and empowerment for all persons. Astronomers can attach a spectrometer to a telescope to determine the
Feminist theory is therefore used to analyze the reasons behind social chemical composition of stars.
inequality.
Beth Richie is an African American feminist and professor. A student wants to present what makes the spectrometer useful to
Her book Arrested Justice: Black Women, Violence, and America’s Prison astronomical research. Which choice most effectively uses relevant
Nation is an important work in the field of feminist theory. information from the notes to accomplish this goal?
Richie’s goal is to expose the ways black women have been left in
particularly vulnerable and dangerous positions. (A) While a spectrometer can spread a wave of electromagnetic
Richie’s research concludes that Black feminist theory provides an radiation into the frequencies that compose it, astronomers are
effective and forceful counter-narrative that can help dismantle the more likely to find its capacity to create a beam of white light
inequality of incarceration. more useful.
(B) Since a spectrometer can use light from a heated object to
The student wants to emphasize the connections between Richie’s goal determine the object’s chemical composition, astronomers can use
and the general idea of feminist theory. Which choice most effectively it to determine the chemical makeup of stars.
uses relevant information from the notes to accomplish this goal?
(C) Astronomers can use a spectrometer to determine the distance that positions.” Moreover, according to the notes, feminist theory is
certain astronomical bodies, like stars, are from Earth. generally “used to analyze the reasons behind social inequality.”
(D) When astronauts take a spectrometer with them into space, they Choice (B) connects these two ideas. choice (A) just states a fact about
can put chemicals from meteorites into the spectrometer to Richie’s authorship. Choices (C) and (D) do not directly connect to
determine the meteorites’ chemical composition. Richie.

Answer Explanations 5. (B) The spectrometer is useful to astronomers because it can attach it to
a telescope to determine the chemical composition of stars—choice (B)
1. (A) The student wants to emphasize a major similarity between mitosis makes this capacity clear. Choice (A) is not supported by the notes
and meiosis. According to the notes, mitosis is used by single-celled since the spectrometer is used to analyze beams of light, not create
organisms for reproduction, and meiosis is used by organisms that use them. It is not choice (C) because the spectrometer is not used to
sexual reproduction. Therefore, both processes are used by living determine distance. It is not choice (D) because chemicals are not
things for reproduction. It is not choice (B) because this shows a inserted into the spectrometer; instead, the light emitted from the
difference between the two. It is not choice (C) because there is no objects is analyzed.
evidence presented in the notes that both processes are used equally
throughout the Earth. It is not choice (D) because meiosis does not
create exact copies of organisms, but instead blends parts of their Additional Practice Exercises
genetic compositions in creating offspring.
The problems in these exercises represent the variety of SAT Writing
2. (D) The student wants to emphasize the type of intervals at which the questions you could encounter. Take about ten minutes for each exercise.
SHC motion will likely oscillate. Choice (D) is the only option that
uses the notes to describe the intervals of oscillation—they are regular Exercise 1
since the notes describe the motion is “periodic.” It is not choice A 1. The European cuckoo mother bird does not randomly deposit her egg;
because the oscillations will be regular, not irregular. It is not choice she chooses the nests of specific species with eggs that resemble
(B) or (C) because these do not describe the oscillation intervals. _______ the European cuckoo. She deposits her egg and leaves it with
the other bird’s eggs, and that is the extent of her parenting.
3. (C) Choice (C) emphasizes the long duration of the bullroarer in human
history by showing that the bullroarer has been around for millennia, Which choice completes the text so that it conforms to the conventions
since the Paleolithic era. It is not choice (A) because this focuses on the of Standard English?
sound of the instrument instead of its duration. It is not choice (B)
because this does not discuss time. It is not choice (D) because it is too (A) those of
vague—we have no clear sense from this just how ancient the (B) these of
bullroarer is. (C) this from
(D) this of
4. (B) According to the notes, “Richie’s goal is to expose the ways black
women have been left in particularly vulnerable and dangerous
2. On certain occasions, he is brutally ashamed of his ______________ Which choice completes the text so that it conforms to the conventions
overcompensates for his treachery by defending his father at all costs. of Standard English?

Which choice completes the text so that it conforms to the conventions (A) professional picture, this
of Standard English? (B) professional picture: this
(C) professional; picture this
(A) father’s deceit, on others he (D) professional picture this
(B) fathers deceit: on others he
(C) father’s deceit; on others, he 5. Do ever you remember hearing in school that the sun—by far the largest
(D) fathers’ deceit—on others, he body in our solar system—is composed almost entirely of the two
smallest elements, ___________________
3. With music that is highly repetitive, Glass has been referred to as a
minimalist and aligned with the work of other composers like La Monte Which choice completes the text so that it conforms to the conventions
Young, Terry Riley, and Steve Reich. of Standard English?
_____________________________ marked by a nonnarrative and
nonrepresentational conception of a work in progress, and represents a (A) hydrogen, and helium.
new approach to the activity of listening to music by focusing on the (B) hydrogen and helium.
internal processes of the music. (C) hydrogen, and helium?
(D) hydrogen and helium?
Which choice completes the text so that it conforms to the conventions
of Standard English? 6. If you choose to teach at the secondary level, it is best to be ready to
answer questions about college, career planning, and young adult
(A) Minimalism a term that Glass has taken strides to distance himself issues. One thing is ___________ a good teacher is there because they
from, is want to be.
(B) Minimalism: a term that Glass has taken strides to distance
himself from is Which choice completes the text so that it conforms to the conventions
(C) Minimalism—a term from which Glass has taken strides to of Standard English?
distance himself—is
(D) Minimalism, a term from which Glass has taken strides to (A) for sure,
distance himself—is (B) for sure: a good
(C) for sure a good
4. When searching for jobs, your online profile should be very thorough. (D) for a
Make sure to post a _________ allows the employer to see that you are
a real person with real skills and makes you more likely to be 7. Detractors argue that the lack of scientific oversight during underwater
interviewed. construction is bound to result in failure. ________ many anticipate the
wealth of knowledge and new discoveries that will certainly follow (D) are both singular, and choice (B) incorrectly uses “these,” which is
underwater monitoring. improper wording in a comparative situation.

Which choice completes the text with the most logical transition? 2. (C) The semicolon in this choice separates the two independent
clauses. The apostrophe before the “s” after “father” indicates singular
(A) And, ownership. The comma after “others” gives a necessary break after the
(B) Consequently, introductory phrase. Choice (A) creates a run-on. Choice (B) does not
(C) Still, show possession. Choice (D) shows possession of plural fathers.
(D) Therefore,
3. (C) The dashes set aside the parenthetical phrase, and the word choice
8. While researching a topic, a student has taken the following notes: in choice (C) is logical. Choices (A) and (B) do not set aside the
parenthetical phrase. Choice (D) uses inconsistent punctuation to set
Pierre-Auguste Renoir was a lead painter in the impressionist movement. aside the phrase.
He was born in Limoges, Haute-Vienne, France in 1841.
The artists Camille Pissarro and Edouard Manet inspired his style and 4. (B) The colon serves to give a needed pause between the independent
painting techniques. clause before the colon and the clarifying independent clause after the
His most popular paintings consist of: Dance at Le Moulin de la Galette, colon. Choices (A) and (D) each produce a run-on sentence. Choice
Luncheon of the Boating Party, and The Swing. (C) interrupts “professional picture.”
Renoir’s paintings are known for their vibrant light and saturated color.
5. (D) The sentence asks a question, so a question mark is needed, making
The student wants to emphasize what makes Renoir’s paintings choices (A) and (B) incorrect. Since just two items are listed, there is
distinctive. Which choice most effectively uses relevant information no need for a comma to separate them, making choice (C) incorrect and
from the notes to accomplish this goal? (D) correct.

(A) Renoir was uniquely influenced by the impressionist artists 6. (B) The colon is appropriate because it indicates a clarification to
Pissarro and Manet. follow. Choice (A) results in a comma splice, while choices (C) and
(B) Dance at Le Moulin de la Galette is one of Renoir’s most famous (D) result in run-on sentences.
works.
(C) The famous impressionist artist Pierre-Auguste Renoir was born 7. (C) This is the only option that provides a needed contrast between the
in France in 1841. argument in the first sentence that the process will result in failure and
(D) Renoir’s impressionist paintings are known for having saturated the current sentence in which there is the potential for a wealth of
color and vibrant light. knowledge and discoveries. The other options do not convey a contrast.

Answer Explanations 8. (D) The notes state that “Renoir’s paintings are known for their vibrant
light and saturated color.” This shows what makes his paintings unique
1. (A) “Those of” correctly represents a logical comparison between the or distinctive. It is not choice (A) or (C) because these do not focus on
nests of the cuckoo and the plural nests of other birds. Choices (C) and
Renoir’s paintings. It is not choice (B) because this does not analyze telephone.
why this painting is famous.
Which choice completes the text so that it conforms to the conventions
Exercise 2 of Standard English?

1. There are more ways than ever to market yourself and your skills, and to (A) brung
network with other professionals in your field. You will want to make (B) brought
sure you are competing in the online job hunt. Career websites like (C) bring
LinkedIn and Monster make it simple to get started. Within your online (D) brang
profiles, it is critical that you _______ information about your
educational background, previous work experience, internships, 4. A spectrometer is an instrument that spreads a wave of electromagnetic
research positions, and volunteer efforts. Now is not the time to be radiation into its component frequencies. When you look through a
modest. spectrometer at a beam of white light, ______________ a continuous
band of colors shifting like a rainbow from red to violet.
Which choice completes the text so that it conforms to the conventions
of Standard English? Which choice completes the text so that it conforms to the conventions
of Standard English?
(A) included
(B) include (A) and you see
(C) including (B) and one can see
(D) to include (C) and he or she can find
(D) you see
2. Newton’s equation explained why that apple fell onto his head,
_______________ and how Earth orbits the sun. It also allowed NASA 5. By creating a metric theory of gravitation, Einstein showed that
scientists to send a man to the moon many years later. phenomena in classical mechanics correspond to inertial motion within
a curved geometry of space-time. This scientific discovery laid the
Which choice completes the text so that it conforms to the conventions groundwork in both astrophysics and cosmology for years to come. Not
of Standard English? only did the theory help to explain an irregularity in Mercury’s orbit,
but _________ and set the theoretical foundations for black holes.
(A) why on the ground one firmly stays,
(B) on the ground one firmly stays, Which choice completes the text so that it conforms to the conventions
(C) why one stays firmly on the ground, of Standard English?
(D) one stays firmly on the ground,
(A) the bending of starlight was also demonstrated by it
3. In his book The Secret Life, Dali inquires as to why in a restaurant when (B) starlight used it to bend the demonstration
he requested a grilled lobster, he was never _______ a boiled (C) it also demonstrated how starlight bends
(D) demonstrating the starlight The Biltmore Estate is the largest privately owned house in the United
States.
6. A popular theory posits that Neanderthals met their extinction through
absorption. That is—supposing Neanderthals were not a distinct The student wants to summarize some of the unique architectural
species, _______ rather a subspecies of Homo sapiens—some features of Biltmore. Which choice most effectively uses relevant
researchers believe that they disappeared after interbreeding with information from the notes to accomplish this goal?
humans when they arrived in Eurasia roughly 80,000 years ago.
(A) The land on which Biltmore sits is approximately 8,000 acres.
Which choice completes the text with the most logical transition? (B) Builders took approximately six years to complete Biltmore.
(C) Biltmore boasts both a conservatory and a bowling alley.
(A) but (D) The climate and scenery of the Asheville area made for a uniquely
(B) and beautiful setting for a large home.
(C) or
(D) to Answer Explanations
7. For those interested in pursuing a career as an environmental engineer, a 1. (B) The text is in the present tense, and “include” is the only option that
bachelor’s degree is a must. _______ a degree in environmental maintains this consistent tense. Moreover, “you include” demonstrates
engineering is necessary, related fields such as general or civil the proper conjugation of the verb.
engineering can be acceptable as well.
2. (C) This is the only choice that is parallel to the other items listed in
Which choice completes the text with the most logical transition? this sentence. Not only are the other options not parallel, they also have
confused word orders.
(A) If
(B) When 3. (B) “Brought” is the correctly used tense of “to bring” in this context.
(C) While
(D) So 4. (D) “Specific” in this case means “unique,” stating that the chemical
signature given off by each element is unique to it. Choice (A) is too
8. While researching a topic, a student has taken the following notes: wordy. Choice (B) is incorrect because “partial to” means “to prefer”
something. Choice (C) uses an adverb instead of an adjective.
Biltmore Estate sits on 8,000 acres of land in Asheville, North Carolina.
It took around six years to complete, starting in 1889 and opening for his 5. (C) This choice is correct because the construction of the second part
family on Christmas Eve in 1895. of the sentence is parallel to the first part of the sentence; additionally,
The mansion has an indoor swimming pool, bowling alley, conservatory, it puts the words in a logical order. Choice (A) uses passive voice.
and library. Choice (B) confuses the intended meaning. Choice (D) is not parallel.
George Washington Vanderbilt II built the estate after falling in love with
the scenery and climate of the area. 6. (A) “But” will logically complete the phrase “not” this, “but rather”
that. The other options would not complete this phrase appropriately.
7. (C) The sentence conveys a contrast between what is necessary and Which choice completes the text so that it conforms to the conventions
what is acceptable. “While” is the only option that provides a contrast. of Standard English?

8. (C) The notes state that “the mansion has an indoor swimming pool, (A) both: the old manner is a lifeless forgery, while
bowling alley, conservatory, and library.” Since these features are (B) both, the old manner is a lifeless forgery while
architectural, citing some of them makes sense if the student wants to (C) both—the old manner is a lifeless, forgery, while
summarize some of the unique architectural features found at Biltmore. (D) both; the old manner, is a lifeless forgery while
The other options focus on nonarchitectural features, like the land area
of the property, how long it took to build, and the setting of the home. 3. In these settings, occupational therapists work with clients to help them
better participate _______ the areas of occupation.

Advanced Practice Which choice completes the text so that it conforms to the conventions
of Standard English?
The following questions are designed to represent the most challenging sorts
of questions you could encounter on the SAT Writing. Take about ten minutes (A) in all the any of
for each exercise. (B) in any all of
(C) in any or all of
Exercise 1 (D) in all of and any of
1. It wasn’t until 1915 that Albert Einstein expanded on Newton’s work to
impart his theory of general relativity, which states that the gravity of 4. One possible justification for involving humans in the actual exploration
any mass curves the space and time around it. Einstein’s theory of is that ___________ increase interest in funding space exploration
relativity is superior to ___________ because it considers special projects.
relativity and can be used when great precision is necessary.
Which choice completes the text so that it conforms to the conventions
Which choice completes the text so that it conforms to the conventions of Standard English?
of Standard English?
(A) they do
(A) Newton (B) it does
(B) it (C) one do
(C) these (D) OMIT the underlined portion
(D) Newton’s law of universal gravitation
5. It can be described as one of the greatest mysteries of modern physics.
2. While using Bernice and Marjorie to model both eras, Fitzgerald finds In a universe that _____________________ neutrality, the apparently
flaws in _____________ the new approach is only relaxed on the asymmetrical distribution of the baryon charge is a singular and
surface. tantalizing puzzle.
Which choice completes the text so that it conforms to the conventions In his construction of an ideal city-state, Plato reevaluates the kinship
of Standard English? (family) relations that comprise society.
Plato sought to replace the kinship system with a new model in which
(A) tends universally, as it were toward net people who aren’t blood relatives still interact in ways now reserved for
(B) tends—universally, as it were toward net familial relations.
(C) tends—universally as it were, toward net Plato argued that the city-state forms a communal family: “every time he
(D) tends—universally, as it were—toward net meets any of them, he will assume he is meeting his brother, or sister, or
mother, or son, or daughter—or the child or parent of one of these.”
6. Panes created using the glass spinning technique are usually identifiable Plato claims that his new model would eliminate the familial loyalty that
for their bulbous centers and wavy surface ____________________ supersedes state loyalty.
the edges joining with the window frame are typically narrower than
the pane’s body. The student wants to present a generalization about how Plato thought
about the family and the state. Which choice most effectively uses
Which choice completes the text so that it conforms to the conventions relevant information from the notes to accomplish this goal?
of Standard English?
(A) Those who have blood relationships to one another should be
(A) texture, additionally they are often very fragile, as excluded from being in the same governmental state.
(B) texture; additionally, they are often very fragile, as (B) Loyalty to the members of one’s family should rightly take
(C) texture. Additionally they are often very fragile as precedence over loyalty to one’s government.
(D) texture additionally, they are often very fragile, as (C) Plato argued that the connections between members of a state
should outweigh the connections from familial relations.
7. Recent studies of Neanderthal anatomy and artifacts suggest that they (D) Kinship is at the core of what will create a monarchical system,
were remarkably well-equipped to deal with the fiercely cold and with the monarchy passed from one generation to the next.
barren conditions, ___________ even thrived within them for nearly
200,000 years. Answer Explanations
Which choice completes the text with the most logical transition? 1. (D) This is the only option that gives a logical comparison between the
theory of Einstein and the theory of Newton. Be sure to compare the
(A) and same sorts of objects to one another.
(B) but
(C) for it was the case that they 2. (A) In choice (A), the colon comes after a complete sentence right
(D) OMIT the underlined portion. before the flaws are clarified and the comma comes before the
transitional “while.” Choice (B) results in a run-on sentence. Choice
8. While researching a topic, a student has taken the following notes: (C) has an unnecessary comma after “lifeless.” Choice (D) has an
In The Republic, Plato attempts to create a blueprint for a just city-state in unnecessary comma after “manner.”
which its constituents prescribe to reason and live in communal harmony.
3. (C) This works because the occupational therapist could potentially be in the same country. It is not choice (B) because this gets Plato’s
help with “any” of the areas or “all” of the areas. Choices (A) and (B) argument backwards. It is not choice (D) because the notes do not focus
do not use prepositions correctly, and choice (D) uses “and,” which on the possibility of a monarchy.
would not make sense because both “all of” and “any of.”
Exercise 2
4. (B) The “it” refers to the act of involving humans in exploration, which
would be singular. “Does” is the correct form of the verb “to do” when 1. Piketty’s data on the wealthy elite makes it somewhat pioneering despite
coupled with a singular subject. Choice (D) could potentially work if its foundations in age-old economics. _____________________
instead of “that” it said “the” immediately before the underlined phrase, redistribution through a progressive global tax on wealth.
and if instead of “increase” it said “increased” immediately following
the underlined phrase. Which choice completes the text so that it conforms to the conventions
of Standard English?
5. (D) This is the only option that sets off the parenthetical phrase with the
same sort of punctuation on either side, i.e., a dash. Commas can also (A) Piketty even offers a solution; economic
set up parenthetical phrases, but there must be a comma before and after (B) Piketty, even offers a solution, economic
the phrase. (C) Piketty even offers a solution: economic
(D) Piketty even offers, a solution economic
6. (B) The semicolon provides a needed break between the two
independent clauses, and the comma after “additionally” gives an 2. As glass enters the bath, its specific gravity and immiscibility with tin
appropriate pause separating the transition from the complete sentence ____________ a continuous ribbon with perfectly smooth surfaces on
that follows. Choice (A) is a run-on sentence, choice (C) lacks a both sides and an even width throughout.
needed comma after the introductory word “Additionally,” and choice
(D) is also a run-on. Which choice completes the text so that it conforms to the conventions
of Standard English?
7. (A) The last part of this sentence gives more support to the claim in the
first part of the sentence, making “and” appropriate. Choice (B) shows (A) causes it to form
contrast, choice (C) is too wordy, and choice (D) removes a needed (B) causes them to form
transition. (C) cause them to form
(D) cause it to form
8. (C) The notes state that “Plato sought to replace the kinship system with
a new model in which people who aren’t blood relatives still interact 3. While Pluto is in orbit around the sun and has become nearly round, it
in ways now reserved for familial relations.” In other words, Plato isn’t big _________________. In other words, Pluto is not
believed that the connections between citizens of a country should be gravitationally dominant in its area and shares its space with
more important than the connections between family members—this astronomical bodies of a relatively large size.
aligns with choice (C). It is not choice (A) because there is no evidence
to suggest that Plato believed that people of the same family could not Which choice completes the text so that it conforms to the conventions
of Standard English?
(A) enough, and therefore doesn’t have enough gravity, to clear its 6. The exchange is notable in that it is the first time Griselda directly
orbit. asserts her desires to Walter, and although she desists as soon as he
(B) enough and therefore, doesn’t have enough gravity, to clear its raises an objection, she allows herself, finally, at _____________ she
orbit. feels to be right and honorable.
(C) enough and therefore doesn’t have enough gravity to clear, its
orbit. Which choice completes the text so that it conforms to the conventions
(D) enough, and, therefore, doesn’t have enough gravity, to clear its of Standard English?
orbit.
(A) what she believes, to be the end of their marriage, to communicate
4. The Parthenon Temple itself was completed in 438 B.C., although to him what
decorative sculpting and engraving within the structure went on for (B) what she believes to be the end of their marriage to communicate
several more years. Since then, the structure has served as to him what
______________________. (C) what she believes—to be the end of their marriage, to
communicate to him what
Which choice completes the text so that it conforms to the conventions (D) what she believes to be the end of their marriage, to communicate
of Standard English? to him what

(A) temple, treasury, church, and most recently, tourist attraction. 7. Rognlie argues that, according to the law of diminishing returns, the rate
(B) temple, treasury church, and most recently, tourist attraction. of return will eventually __________ on to say that Piketty has an
(C) temple treasury, church and most recently tourist attraction. inflated idea of current return and doesn’t consider depreciation.
(D) temple treasury church, and most recently tourist attraction.
Which choice completes the text so that it conforms to the conventions
5. Strindberg and Ibsen—writing in the same genre at the same point in of Standard English?
history and emerging from both the same level of society and corner of
the world—_____________ developed remarkably antithetical (A) decrease; goes
worldviews, each powerful enough not only to weather the criticism of (B) decrease, goes
the opposition but to develop and grow despite it. (C) decrease; he goes
(D) decrease, he goes
Which choice completes the text with the most logical transition?
8. While researching a topic, a student has taken the following notes:
(A) nonetheless
(B) consequently Cosmic expansion is the idea that an expanding universe could be traced
(C) also back to a single point.
(D) divergently The scientist Georges Lemaître hypothesized the idea of cosmic expansion
in 1927.
In 1929, scientist Edwin Hubble confirmed Lemaître’s work when he
discovered that galaxies were drifting apart.
For many scientists, Hubble’s observations gave convincing evidence of the surrounded by two dashes or two commas to separate it from the rest of
big bang theory. the sentence. Choice (B) uses the two commas but splits the
The big bang theory states that after cosmic expansion, the universe cooled parenthetical element into pieces. Choice (C) omits the first comma.
to form subatomic particles. Choice (D) has entirely too many commas within the parenthetical
The theory also states that these subatomic particles that created atoms and element, preventing continuity. Choice (A) is the best option.
the elements.
These elements went on to form stars and galaxies. 4. (A) This choice gives necessary breaks between all the listed items and
has a break after the clarifying phrase “most recently.” Choices (B),
The student wants to highlight the connection between the work of (C), and (D) all change the original meaning because of their comma
Lemaître and Hubble. Which choice most effectively uses relevant placements or lack thereof.
information from the notes to accomplish this goal?
5. (A) “Nonetheless” means “in spite of,” which makes sense given the
(A) Hubble provided experimental observations to support Lemaître’s fact that these writers had similar backgrounds yet ended up having
hypothesis. very different worldviews. “Consequently” in choice (B) indicates
(B) Lemaître used the observations that Hubble made about the big cause and effect. “Also” in choice (C) indicates the continuation of
bang to formulate a comprehensive scientific theory. thought. “Divergently” in choice (D) could apply toward the
(C) Modern scientists greatly appreciate the detailed observations that differences in their views but does not work as a transition to show a
both Lemaître and Hubble made in the 1920s. logical contrast.
(D) Lemaître’s theory about subatomic particles was confirmed by
Hubble’s theory about the big bang. 6. (D) The entire phrase “at what she believes to be the end of their
marriage” is parenthetical. Choice (D) is the only option that both
Answer Explanations leaves this phrase intact and places a comma at the end of it so that it is
set aside from the rest of the sentence.
1. (C) This option correctly uses a colon to set off a clarification. Choice
(B) uses a comma, which does not provide a sufficiently significant 7. (C) This option both clarifies the subject and uses a semicolon to give
pause. Choice (D) has a pause in an awkward spot. Choice (A) needs a a clear break between the independent clauses. Choice (A) does not
complete sentence after the semicolon. have the necessary independent clause after the semicolon. Choice (B)
does not give a parallel construction. Choice (D) creates a comma
2. (D) The compound subject of “specific gravity and immiscibility with splice.
tin” demands a plural verb; hence, “cause” works. Also, “it” is correct
since the pronoun refers to the singular “glass.” 8. (A) The notes state that Lemaître hypothesized the idea of cosmic
expansion, while Hubble made observations that confirmed this theory.
3. (A) The parenthetical element here was tricky to diagnose. It is not choice (A) because Lemaître preceded Hubble. It is not choice
Nonetheless, “and therefore doesn’t have enough gravity” was the (C) because this does not explain how Lemaître and Hubble are
parenthetical element; remove it from the sentence and the clause still connected. It is not choice (D) because there is no evidence that
functions perfectly well. As a rule, a parenthetical element can be Lemaître had a theory about subatomic particles.

You might also like